You are on page 1of 89

T-Tail advantage &

disadvantage ?!

Hi,
Could you explain me
the good & bad sides of
a T-Tail airplane like the
BAe 146 ? and also your
opinion about that
config.
It would really clarify
some discussion ........
ThanX
R@y

9th February 2004, 04:02

Intruder
Join Date: May 2000
Location: USA
Posts: 1,707

9th February 2004, 04:59

#2 (permalink)

Pro: Horizontal stab is in "clean" airflow in virtually all flight


regimes (deep stall excepted, but uncommon in air carriers).
Con: Vertical stab structure must be stronger to carry the loads.

#3 (permalink)

G-ALAN

One advantage is it's not likley to be hit and damaged by anything


thrown up by the tyres, especially on a rough strip or gravel strip.
bat fastard
I fly a T tail and one thing I find very annoying is the fact I can't
see the top of the elevators on the walk around. Also T tails don't
Join Date: Apr 2003
perform quite as well in stalls, infact if the stall is deep enough
Location: Back home in Alba
there is a risk you won't have enough elevator authority to
Posts: 132
recover.

9th February 2004, 22:01

safetypee
Join Date: Dec 2002
Location: UK
Posts: 693

#4 (permalink)

A T Tail increases the elevator effectiveness by giving a larger


effective span. High elevator effectiveness is required for high lift
wings, which are often mounted above the fuselage, thus the tail
has to be out of the down wash. The end plate effect also gives
more rudder effectiveness for a shorter tail arm. i.e. YC-14/15,
C17, Dash 6/8, and 146/RJ.

The 146 still has the highest Clmax for any civil airliner. Deep
stalls? For the 146 in theory yes, but the test team tried very hard
to find it yet never experienced any during development. The stick
push gives protection, but during certification it was also a quick
method of determining the expected stall speeds so that
performance testing could progress quickly.

#5 (permalink)

9th February 2004, 22:10

747FOCAL

G-ALAN,

Join Date: Aug 2003


Location: Skagness on the
beach
Posts: 884

Not sure I agree with you on the t-tail and stalls. Maybe in
something like an MD-80 where the designers put the wings
closer to the tail than they should have, but not on something like
a 727. I have also heard wind up stalls can be a
beeeaaaaattcchhhh in a t-tail.

10th February 2004, 08:25

G-ALAN
bat fastard
Join Date: Apr 2003
Location: Back home in Alba
Posts: 132

10th February 2004, 09:59

DanAir1-11
Join Date: Sep 2003
Location: Amidst the dust
and the flies, somewhere in
Western Australia
Posts: 98

#6 (permalink)

Ok I stand to be corrected I've always been taught low wings


and a T-tail are not a good combination in a stall as turbulent air
from the wings will flow over the elevators thus giving them little
authority.

#7 (permalink)

On the subject of T-tail stalls. A stall situation in a T tail is neither


more nor less critical than that of any other configuration,
providing it is recognised at the onset and recovery is effected
promptly and correctly. The real problem occurs when the stall is
allowed to develop into a deep stall, from which many aircraft
have been lost.
Two notable examples being a BA Trident over Staines UK in the
60's where the PF retracted the leading edge 'droop' devices too
soon leading to aerodynamic deep stall at a fairly low altitude,
and a Northwest 727 on a ferry flight, where the young crew were
mislead into the stall after pitot tubes became iced and
subsequently blocked, the crew became aware of high airspeed in
the climb, which they attributed to having a light aircraft. They
pulled her back and let her climb, even mistaking the onset of the
stall buffet as Mach buffet. As in the first incidence, the entered
the deep stall and were unable to effect recovery.
The vast majority of my experience has been with T tails, 1-11's
which were a pleasure to fly, as was the 727. THere have been

many things put in print about tyhe pros and cons of the
aerodynamics of the design, If I don't forget, i will come back to
this thread and add some links later on.
Regards

#8 (permalink)

10th February 2004, 14:05

Loose rivets
Psychophysiological entity
Join Date: Jun 2001
Location: Hmm I broil a
winnable
Age: 70
Posts: 2,239

The deep stall of Mike Lithgow's 1-11 comes to mind.


DanAir 1-11, were u there when CC and CP turned up? I recall
that all the training was done on the real a/c...the stall through to
the push. Not sure that i would want to do that now.

#9 (permalink)

10th February 2004, 15:50

DanAir1-11

Loose rivets,

Join Date: Sep 2003


Location: Amidst the dust
and the flies, somewhere in
Western Australia
Posts: 98

That doesn't half bring back some memories!.


Was PNF on the famous phantom engine fire flight out of
Manchester on AXCP (BAC 1-11 401AK - build# 87 for those
confused by this) , she certainly earned her "smoke-bomb"
handle! I - Re the stall training, never had the pleasure at that
time, and i'm fairly sure that was a good thing!.
As I recall Bryn Wayt did something similar?

#10 (permalink)

10th February 2004, 23:17

Volume
Join Date: Dec 2001
Location: what U.S. calls
old Europe
Posts: 355

Greatest advantage of the T-Tail is, that is far obove those well
trained service vehicle drivers, and therefor not hit by catering
trucks, stairs, fuel trucks etc. which damage conventional tails
nearly every day on some airport on this planet.

#11 (permalink)

11th February 2004, 10:21

DanAir1-11

volume,

Join Date: Sep 2003


Location: Amidst the dust and the
flies, somewhere in Western
Australia
Posts: 98

ain't that the truth!. Was out at a dirt strip last weekend
and it was fairly warm, somewhere in the region of 48
degC open shade
Ramped next to a Cessna 'something' and a not so bright
chap decides that he will park his ute (pick-up) under the
wing in the shade, whilst working on her. Only one

problem, he apparently forgot about his drum of kero on


the tray complete with pump sticking out of the top end a
proceeded to remove the stbd flap assy and puncture just
forward of the mounts! Think he did most of the damage
reversing attempting to extricate himself.

#12 (permalink)

12th February 2004, 02:36

BigGreenPleasureMachine Dan Air makes an important point, when the tail is


Join Date: Jul 2000
Location: 1060 West Addison
Posts: 67

unaffected by the down wash from the wing, natural stall


warning doesn't exist. Hence the stick shakers/pushers on
aircraft of these configurations, which works well as the tail
won't be in the turbulent air at the stall warning speed.
A more problematic characteristic is the tendency for t-tails
with aft mounted engines to pitch up on stalling, making
things worse if the tail has already become ineffective. i
imagine the abscence of this quality on the 146/RJs is to
do with their engine installation position.
Regards, BGPM.

#13 (permalink)

12th February 2004, 03:08

Tinstaafl
Join Date: Dec 1998
Location: Escapee from
Ultima Thule
Posts: 2,930

I've often considered what one could do to try to exit a deep stall.
I think there's two or Given enough height I reckon I'd try:
1. Stick out all the low speed devices
2. Have everyone down the back see how close to the cockpit
they can get + gear down to try to lower the drag line.
3. combined cycling power & elevator to try to destabilise the
thing. May not be all that effective if there's not much power
induced pitching.
4. Try to roll using aileron & spoilers far enough to allow gravity &
airflow to cause a yaw towards the ground ie into the airflow.
5. Also try to roll, but using asymmetric power & rudder. Risk of
spinning but at least there's the chance of an unstable bit while
transitioning.
Discussion? Alternate suggestions?

12th February 2004, 08:27

#14 (permalink)

DanAir1-11

Best remedy for a deep stall?

Join Date: Sep 2003


Location: Amidst the dust
and the flies, somewhere in
Western Australia
Posts: 98

Don't let it get there in the first place.

#15 (permalink)

12th February 2004, 19:45

Field In Sight

You could also fit retro rockets near the cockpit facing upwards.

Join Date: May 2002


Location: Sale
Posts: 245

a la The Space shuttle.


FIS.

#16 (permalink)

12th February 2004, 20:37

ft
Join Date: Oct 2000
Location: N. Europe
Posts: 372

Suggested method to get some jet fighters out of a deep stall is


rocking it out ( \m/ ) of it. The elevator authority remaining
wont allow you to get the aircraft out, but you can get it to sway
around the stable null point. Keep feeding the oscillations with
elevator input, and eventually the nose will be low enough on the
downswing to let you get out of it. Theres a manual override on
the elevator control in the F-16 to enable you to do this. Im
hoping for someone to fill in on the exact name of the switch and
just how the elevator works in that mode.
Stick pushers might provide a stall warning, but if that was their
only purpose theyd still be stick shakers. Keeping you out of a
deep stall is the thing which makes them a required item.
IMO, T-tails were largely a fashion fad, along with rear-mounted
engines. Those two sorta go hand in hand anyway.
An advantage yet to be mentioned is that the elevator wont blank
the rudder in a non-inverted spin where having a bit of rudder
authority just might come in handy.
Con is structural, having all that weight up there on that long
moment arm. Look at gliders groundlooping, it is not uncommon
for the fuselage to snap due to the torque put on it by the T-tail.
OTOH, T-tails are generally preferred due to the fact that landing
out in crop will have a good chance of damaging the elevator of a
non-T stab severely.
G-ALAN, of course you were right.
Cheers,
Fred

12th February 2004, 21:07

Bre901
Self Loathing Froggy
Join Date: Jun 2002
Location: elsewhere
Age: 4
Posts: 547

#17 (permalink)

Slightly O/T but


still about flying
machines
For gliders it is
more than a
fashion : since the
Phnix (1959),
90% of plastic
gliders have been
designed with Ttails, notable
exceptions being
the Std-Libelle
and pure aerobatic
gliders (for
structural reasons
as mentionned by
ft).
The determining
pros are indeed
the interaction
with crops and the
rudder authority in
spins. Gliders are
probably the most
stall & spin prone
airplanes
(aerobatics
excepted) as
people tend to fly
as slow as
possible (min sink
or even less)
when cirling in
thermals.
Never heard of
deep stalls for
gliders. (or seen
anything in flight
manuals). I guess
the layout is not
favourable
(forward position
of wings)

Eyelets on upper wing of Airbus

Well, at least I think it was an airbus.


I was on a flight the other day with our CEO, who loves aviation. I could
answer most of his questions, but got stuck when he pointed out the
yellow painted eyelets on the upper wing. Didn't look quite big enough
to be hoisting points.
Anyway, I need to correct my otherwised unblemished perceived
aviation-knowledge with an answer, sad as it may be.

15th December 2002, 23:45

woderick
Join Date: Apr 2002
Location: IO83VI
Posts: 130

16th December 2002,


00:30

paulo
Join Date: Aug 2000
Location: London
Posts: 678

18th December 2002,

#2 (permalink)

So if it was the
upper wing you
were obviously in
a biplane - else
they were the
hoisting points on
the upper wing
surface ( of a
monoplane) used
for hanging the
aircraft from the
hangar roof during
maintenance else they are
anchor points for
slide rafts should
such things ever
be needed during
evacuation.
see previous
threads and Take
your pick[b]
[color=red]

#3 (permalink)

yeah yeah - i was using the term 'upper wing' for brevity, so these
eyelets were on the (sorry to have to clarify) monoplane airbus
variants

#4 (permalink)

20:55

Airbus.De

Also a safty harness point for maintenance Lads/Ladies

Join Date: Oct 2002


Location: Hamburg
Posts: 37

19th December 2002,


23:08

boris
Join Date: Oct 2000
Location: UK
Posts: 112

23rd June 2003, 00:05

galley-wench22
Join Date: Jun 2003
Location: LBA
Posts: 67

#5 (permalink)

Having just read the interesting thread about the increased TOW
available with a more rearward C of G on the 73, I expect that the
ever-practical Airbus people have fitted these eyelets for the extra
lift available with said rear C of G!

#6 (permalink)

I work on the 733 and the eyelets are on the wings of my aircraft
too.
A cord with a hook can be found within the frame of the overwing
exits. When attached to the wing they can be used as a guide to
hold onto during a evacuation after a ditching.
However, there is nothing in our ditching drill about using them. It
is just one of those little extra things if time/cercumstances
permit them to be used.

23rd June 2003, 07:30

used2flyboeing
Join Date: Jun 2003
Location: las vegas
Posts: 110

#7 (permalink)

I heard these were clever recessed vortex generators - IE AIRBUS


loves to crow about their clean wing designs as opposed to
Boeing's love affair with dispatch critical VGs on wing, nacelles &
empennages ..
I find it hard to believe that a stew is going to take the time to
hook ropes into these things following an unscheduled incident ....

24th June 2003, 18:02

idg
Join Date: Jan 1999
Location: hongkong
Posts: 185

#8 (permalink)

I think you will find that these hard points are for use if the slide
deploys but then deflates.
In these circumstances there is an attachment hook on the slide
that can be attached to this hardpoint and allow someone to
tension the slide from the ground, thereby allowing it's continued

use.

#9 (permalink)

26th June 2003, 05:32

SOPS
short flights long nights
Join Date: Aug 1999
Posts: 727

No slides on a 73 or A320 over the wing, they are to hook in a


"help" rope. Maybe you big guys in HongKong have slides over
wing, but not us!!

#10 (permalink)

27th June 2003, 05:39

eyeinthesky
Join Date: Feb 2001
Location: Hants, UK
Posts: 1,036

I thought they were crane attachment points. They're the only


way you'll get a decent rate of climb out of an A320 or A340!!

#11 (permalink)

29th June 2003, 15:30

Filtonman
Join Date: Aug 2002
Location: Wales
Posts: 19

I used to design A319/20/21 wings in a previous life...they are


indeed life rope attachment points (or drowning rope when the
aircraft sinks).
Next pub quiz question what are the large sticky out bits at the 2
O'clock ish position on A320 engines for?
Clue ..it relates to high AOA flight.

#12 (permalink)

5th July 2003, 02:15

used2flyboeing

I know ! I Know !! Ooo! Ooo! I Know ! I Know !

Join Date: Jun 2003


Location: las vegas
Posts: 110

Im not sure what you mean by Sticky out bits - but,


You must be referring to the protrusions on the engine nacelles IE Strakes or Chines in US vernacular.
These have been used for several reasons - Douglas used two of
them per nacelle on the DC-10 to increase lift ( Kerker was the
Douglas Engineer that did the development ) - Im not sure of the
aerodynamic theory as to how they work - because of the engine
nacelle coupling - IE the DC-10 engines hang really low ( A300
has literally DC-10 engines, nacelles & struts ) - and therefore the
nacellel vortexs do not interfer with the flow over the wings &
spoil lift. Boeing & AIRBUS use the nacelle chines for a different

purpose - to improve low speed stall characteristics- they


physically improve the strength of the nacelle vortex so that it will
not burst as it passes through the leading edge pressure gradient
- these are vortex shed amplifiers - otherwise the
characteristically week vortex would "burst" as it passes though
the pressure differential when going from underside to topside of
the wing. A bursting vortex will hose the flow over the top side of
the inboard wing causing it to want to stall before the outside
wing. Because of the fact that modern jets use a swept wing - this
stalling unloads the inboard wing first - while the outbord reagion
is still flying. This can cause a stall pitch down moment - that will
limit your slowest approach speed - IE airplane will keep pitching
over following stall & brealk - not allowing you to continue slowing
down with AOA. If you make the damn thing too effective ( IE the
size of the chine ) you will make the inboard wing hang on too
long- IE NOT want to stall at all - making the aircraft divergent in
pitch - IE Airplane wants to stay pitched up regardless of what the
pilot wants - SSSoooooo - as St Augistine said - prudence is in the
middle - IE there is a sweet spot in between too small & too
Large. AIRBUS copied this use from Boeing - who did it on the
737, 767 & 777. The A320-100 delivered with out these & put
them on later after they considered the cost & expense of trailing
edge improvements. Now everybody uses them Embraer ERJ170
included. THe big question is WHY AIRBUS uses two of them
instead of Boeing's use of one - & that is because they do not
want to spare a left & right handed nacelle - IE they have a
universal part. I believe Embraer is subscribing to that line of
thinking ..
Correct ?

5th July 2003, 15:52

john_tullamarine
Moderator
Join Date: Apr 2001
Location: various places .....
Posts: 3,768

5th July 2003, 18:07

javelin
Join Date: Apr 2000
Location: 30 West
Age: 51
Posts: 887

#13 (permalink)

... to do with larger engines and reduced nacelle-wing clearance


on later aircraft causing nacelle vortex flow to go over, rather than
under, the wing. The chine (big VG) makes the flow a little better
behaved at high alpha.
There is a good article in the Tech Log URL sticky which gives an
overview of what is going on....

#14 (permalink)

As memory serves, the chines are only on the CFM's, not on the
V2500 on a 320. Tere again, how much notice do I pay on a
walkround, I'm too busy dodging people who want to fine me for
not wearing a vest

6th July 2003, 01:08

used2flyboeing

#15 (permalink)

Anyone know for sure why AIRBUS uses 2 of these on A319 &
A318 ??

Join Date: Jun 2003


Location: las vegas
Posts: 110

8th July 2003, 09:30

idg

#16 (permalink)

And on the IAE 321 actually. Don't know about the CFM.

Join Date: Jan 1999


Location: hongkong
Posts: 185

11th July 2003, 22:23

#17
(permalin
k)

allthatglitters
Join Date: Oct 2001
Location: farrrr east
Posts: 257

" but got


stuck
when he
pointed
out the
yellow
painted
eyelets
on the
upper
wing."
For the
A320,
these are
anchor
points for
a rope
stowed in
the
narrow
doored
over
head
stowage,
around
the

o/wing
area,
have
noticed
that the
ropes are
not on all
operaters
aircraft.
I'll follow
up later.
And yes
the B737
Classic
has the
rope in
the top
corner of
the cut
out for
the
O/wing
hatch.
HSI: Track up vs Heading up

I've never used a Track up oriented HSI.


I'm told it can be disorientating if you are not used to it.
When you turn to a heading in Heading mode, will you have the selected heading
at 12 o'clock?
I don't understand

24th May 2003, 03:15

Notso Fantastic
Join Date: Aug 2000
Location: UK
Posts: 1,814

#2
(permalin
k)

Yes, the
compass
is bang
on your
heading
with
current
heading
bang on
12
o'clock.
That's
the way
nature

intended
it! Never
flown
track up.

#3 (permalink)

24th May 2003, 03:52

LEM
Join Date: May 2003
Location: The Roman
Empire
Posts: 801

I agree, that's why I'm asking...


Check this page :
http://www.b737.org.uk/flightinsts.htm
You can notice a TRK UP HSI on a 737 classic.

#4 (permalink)

24th May 2003, 16:11

BOAC
Per Ardua ad Astraeus
Join Date: Mar 2000
Location: UK
Posts: 11,172

24th May 2003, 16:24

4Greens
Join Date: Jul 2000
Location: sydney
Posts: 469

24th May 2003, 18:48

Notso Fantastic
Join Date: Aug 2000
Location: UK
Posts: 1,814

24th May 2003, 22:11

I cannot recall why the option is there - I was told once (Tempus
fugit) but I'm sure there is a good reason and someone will offer
it soon!
I'm confused enough as it is

#5 (permalink)

Heading is not at 12 the track is. It will only be at 12 if there is


zero drift. Track makes instrument approach procedures
extremely sensible, as track is what you are supposed to fly. Have
never known anyone not like it. There is no disorientation.

#6 (permalink)

I see the confusion! I don't like it. To me, compasses are to tell
you which way you are looking. Hence HDG. Tye explanation
under 'Map Mode' explains that the diagram is to represent that
some people may want an oddball representation of TRK up and it
takes a software change to allow TRK up. I would give it no more
thought. I shall have a play tonight on the 747-400 and see if it is
possible to have it that way but I'm fairly certain it can't even be
done (not that you would want to- you have a very effective white
TRK line). I reckon that diagram is just to show you that it can be
accomplished (with software changes). Give it no more thought
other than to the option of HDG (True) (long overwater/polar
flights) and HDG (Mag).

#7 (permalink)

KingoftheRoad
Join Date: Jul 2002
Location: here & there
Posts: 68

bmi(baby) 737's are indeed 'track up', but their Airbus fleet is all
'hdg up'.
When converting from one to the other, nobody even thinks to
mention it - you just set the hdg. req'd. & read the trk., or set the
trk. and the hdg. is what it is, depending on the wind.
Much easier on the 'bus using the 'flight path vector', then you
can fly any trk.you wish, and forget about the drift.,much more
useful, v.clever.
Roger Miller.

#8 (permalink)

25th May 2003, 12:04

QAVION
Posts: n/a

"I shall have a play tonight on the 747-400 and see if it is possible
to have it that way but I'm fairly certain it can't even be done (not
that you would want to- you have a very effective white TRK line).
"
There is no pilot selection for this feature. It is, however,
selectable through a combination of wiring (pin programming) and
software changes on aircraft like the 747-400. Some simulators
which are used by a number of airlines may allow this selection
with a push of a button (not found in a normal aircraft).
Our 747-400 and 767 fleet has "TRACK UP" for whatever reason
(perhaps because we are generally a long haul airline where most
of the time we are following an LNAV track). Most airlines, I
believe have a HDG UP display. KLM, I seem to remember,
switched from one to the other.
Regards.
Q.

25th May 2003, 13:25

#9 (permalink)

john_tullamarine

For the 733 etc, flying track up on the EHSI makes letdowns a
doddle .. one does the normal brain strain nav solution guesswork
and starts out with a best guess heading on the heading bug and
Join Date: Apr 2001
the track display tells you the TMG ... corrections to track are a
Location: various places .....
breeze and flying an ILS to the ground was never so easy ....
Moderator

Posts: 3,768

Being the dinosaur that I am, I resisted it for a while .. but it


didn't take too long before the simplicity seduced me ...
At the end of the day it is still a preference sort of thing, subject
to a particular operator's SOP directions, of course ...

#10 (permalink)

26th May 2003, 01:59

Intruder
Join Date: May 2000
Location: USA
Posts: 1,707

On our 744s the Nav Display is Track Up, but the compass card on
the Primary Flight Display is Heading Up. Neither are selectable in
the cockpit (only True/Magnetic is selectable).
As with virtually all airborne systems that have a couple primary
options, there are good and bad points about either one.
When "following the magenta line" with an FMS, Track Up makes
all kinds of sense, because you can reference the wind-corrected
heading at a glance (it's at the top!). No mental gymnastics are
required to manually fly a complicated Departure Procedure
(SID), even without a Flight Director.
OTOH, the Heading Up display is a good quick reference in high
crosswinds to get a mental picture of your drift, and subsequently
align your eyeballs to find traffic or the approach lights.

#11 (permalink)

27th May 2003, 15:10

planecrazi
Join Date: Oct 1999
Location: In the oil wealth
of sand dunes
Posts: 277

In our fleet of A340's, only one has track up. This you cannot
select and comes with the software. Others are all Hdg up.
Unusual to have one black sheep in the fleet when they all come
from the factory.
Track up:-green line in twelve o' clock position on ND with nose
off centre due to drift. (if drfit)
Hdg up: Heading in twelve o' clock with green line running off
centre due to drift.(if drift)

#12 (permalink)

30th May 2003, 08:54

Barbers Pole
Join Date: Jul 2000
Location: New Zealand
Posts: 137

All our 733 are track up.. Once you get used to it hand flown
dep/app's are a piece of p#ss. eg, "airbourne turn left trk 115" so
take off roll left until the trk line says 115, no drift guess's req'd.

NDB app, "cross" the beacon turn the until the trk line is the same
as the FNA crse check raw data on RMI
Holding patterns the same.

30th May 2003, 15:46

LEM

#13 (permalink)

Yes, but I can imagine your track changes when the wind
changes, so it's a half step forward.

Join Date: May 2003


Location: The Roman
Empire
Posts: 801

31st May 2003, 11:32

Bundi Gap
Join Date: Dec 2002
Location: Australia
Posts: 2

31st May 2003, 13:42

Notso Fantastic
Join Date: Aug 2000
Location: UK
Posts: 1,814

The real one is on the 777, where you can select to command a
certain track with the heading knob , which then becomes a true
track knob .
Am I wrong?

#14 (permalink)

Generally Boeing aircraft have nav displays with track up and


Airbus aircraft have NDs with heading up. I have flown both once you get used to the different display methods they both
seem to work ok.

#15 (permalink)

Seems to me all the


advantages stated for 'Trackup' are on a 'Hdg-up' display
too. On the 747-400 fit, with
'Hdg-up', you have a white
track line which will usually be
offset to the left or right
(depending on drift) of your
Hdg-up line. Therefore to fly
approaches or go direct
anywhere, just put the white
line on the final approach line
or place it on your direct-to
position, and Bob's your uncle.
Compasses are meant to show
where you are facing. Any
nautical type will know that,
and try and allow for drift
(tide) mentally. 'Twas how it
was meant to be. Track-up is a
mite too clever for itself- for
instance, when you stop
taxiing, what does the
compass display do? Does it
swing out of control because
essentially, by definition, you
have no track and it is not
telling you anything then, or
does it default to something
sensible-ie. Hdg-up? Breaking
out of cloud near 2 airfields
with similar runways, Hdg-up
will immediately tell you which
is the wrong one-Track-up
could actually confuse you as

the Runway nomination may


not match the way the
compass says you are
Heading.

31st May 2003, 14:05

dvt
Join Date: Apr 2002
Location: Lands End
Posts: 55

#16 (permalink)

Track up is an excellent
situation awareness tool. If
you're proceeding direct,
"screw the heading bug",
simply roll out with your Track
Line over you fix point. The
drift is automatically killed.
Can it get any easier?!
Hell, roll out on final and put
your track line through the
runway symbol and you're
done with wind adjustments
for a non-precision approach.
Can it get any easier?! In
heavy x-winds, in the weather,
with your bug on runway
heading you can train yourself
to where to look for the
runway when you pop out of
the clouds. ie Your eyes need
to go in the same direction
you need to go to place the
heading bug over the Track
Line. This is why, I believe,
Boeing has the heading bug
jump to runway heading at
LOC Capture. Their actually
helping pilots out, by
designing systems that think
like like pilots. Can it get any
easier?!
Also when manuevering for
thunderstorms, simply
displace your Track Line
relative to the storm. It's a
snap. The Track Line has
range gates or tick marks that
make displacement a nobrainer. You can use your
fingers as dividers, or just
WAG it. Can it get any
easier?! It's a bit funny to

watching former Airbus guys


dogging T-storms with the
heading bug. They're not
clued into the fact that their
Track Line is going directly
through that big Red Cell up
ahead. Look out the window
and tell me what's wrong with
this picture.

The Trap in the MAP on ILS

Some operators require both pilots to stay on MAP mode (B737 Classics)
during an ILS because the localiser and glide slope displacement is readily
seen on the ADI. Others allow a choice between HSI mode and MAP mode.
On ILS with flight director, we know that the ADI localizer automatically
goes to expanded scale. That's fine. Now try this in the simulator: Set up
the ILS as above but on MAP mode. Ensure IMC.
Near the outer marker, deliberately turn the aircraft say 10 degrees off
track (to simulate incompetent handling) and note that very quickly the
expanded mode needle on the ADI goes to full deflection and off scale.
Stay on glide slope while doing this.
Beyond full scale ADI expanded localiser you now have no idea how far you
really are off the centre line. Only HSI mode will show you that. Looking at
the MAP mode will not tell you how many dots you are off centre.
After several seconds of flying this offset heading from the ILS centre line,
freeze the simulator and switch from MAP to HSI mode. By now after say
20 seconds you will see that the aircraft is beyond full scale normal
localiser and heading for potential disaster depending on terrain.
Now switch the simulator to CAVOK and note the runway way off to the left
or right although you are still on GS.
What is my point, you are entitled to ask. It is this.
If you inadvertently deviate from the ILS centre line for whatever reason
(drift or inaccurate flying) then once the expanded localiser reaches full
scale for just a few precious seconds you lose awareness of just how far
you have deviated unless you quickly switch to HSI mode and check the
number of dots deviation.
You can go completely off the HSI localiser scale -yet the expanded
localiser on the ADI will still stay expanded and all you know is that you
are currently more than just one dot off centre. In other words the
expanded localiser needle on the ADI is now useless.

The MAP mode certainly may aid situational awareness in terms of


positions of the runway centre line and of other information such as missed
approach track, but there is no way you can judge the number of dots off
track merely by scanning the MAP.
And it is the number of dots off track that dictate whether or not the
deviation is serious enough for an immediate go-around. And only the HSI
mode will give you that information. For this reason I believe that for an
ILS the HSI mode is safer in the long run than MAP. Having said that I am
aware that Boeing recommend MAP mode for an ILS.
And hands up all those who have never deviated badly off the ILS centre
line on an ILS?

#2 (permalink)

2nd June 2003, 02:50

bookworm

Hudson

Join Date: Aug 2000


Location: UK
Posts: 2,949

For those of us unfamiliar with


the aircraft or display setup,
could you indicate:
1) lateral distance or angular
displacement indicated by FSD
on the ADI
2) lateral distance or angular
displacement indicated by FSD
on the HSI, and by one dot on
the HSI
3) the scale of the map as
depicted in MAP mode
?
Thanks

#3 (permalink)

2nd June 2003, 04:33

None
Join Date: Jul 2000
Location: West
Posts: 364

Same for intercepts

I had always thought it was necessary (and prudent) to use raw


data for the capture. However, during my last CQ training, I was
instructed it was not necessary as the expanded mode provides
sufficient information (and Progress page 2 gives offset from an
FMS calculation). Since company standards don't specify what
mode to be in for ILS approaches, and after thinking about your
post, it may not be a bad idea to remain in raw data.

#4 (permalink)

2nd June 2003, 08:51

stillalbatross

I thought the 737 classic had steam driven dials. Which ancient
737 are you talking about?

Join Date: May 2003


Location: 'round here
Posts: 316

#5 (permalink)

2nd June 2003, 12:19

QAVION
Posts: n/a

"You can go completely off the HSI localiser scale -yet the
expanded localiser on the ADI will still stay expanded and all you
know is that you are currently more than just one dot off centre. "
Is your F/O also situationally unaware?
Is this typical display behaviour? Do most glass cockpit aircraft
types latch the MAP displays in the expanded mode, or do the
displays go back to normal if you go out of range of the expanded
display? (say, 1.25 dots)
I know there are a few differences when it comes to this sort of
thing. As far as I know, the 747-400, for example, only produces
an expanded display when the LOC is engaged (so there will be
less chance of you going off the ILS beam, once established
thereon).
Regards.
Q.

#6 (permalink)

2nd June 2003, 17:47

LEM
Join Date: May 2003
Location: The Roman
Empire
Posts: 801

Stillalbatros, 737's are called "original" = 100 & 200


"classic" = 300, 400 & 500, with ADI - MAP EFIS
"NG" full EFIS.
In the 744 precision simulator at least, if you deviate too much,
the scale turns into normal again.
My brother's company sop calls for the opposite as aerobat's one.

2nd June 2003, 22:01

Hudson
Posts: n/a

#7 (permalink)

Bookworm. Not quite sure what you mean by the FSD but assume
it is the localiser and glide slope info on the ADI and HSI?
Normal LOC deviation is one degree per dot. When VOR/LOC
engaged (not necessarily on auto pilot either) the ADI scale

automatically expands to indicate half a degree deviation per dot.


It remains expanded until after landing roll-out or on a go-around
with RA greater than 200ft.
There is no distance lateral deviation indication on either the
expanded ADI scale or HSI although you could work it out by
mental arithmatic if you knew the exact distance from the
localiser aerials. Forget that for a joke. But as an example if you
are 5.486 miles from the threshold then one dot on the normal
scale on the HSI means you are 774 feet off centre line.
If you fly the ILS without the flight director then the ADI localiser
scale is the same as the HSI - that is no expansion. In that case if
you are on MAP mode for situational awareness it doesn't really
matter because the ADI localiser gives you all the information
necessary to tell you exactly how many dots you are off the
centre line.
Put another way - if you allow the aircraft to stray off the
centreline full scale on normal HSI mode then it is an immediate
go-around situation. But if you are on MAP mode and relying on
the expanded scale on the ADI to keep you within limits when on
flight director, then you are whistling into the wind
Of course a brilliant PNF would quickly advise the PF that he had
deviated off scale on the expanded ADI when on flight director
coupled to ILS. But the moment that the PF allows the aircraft to
drift off beyond full scale on the expanded localiser you are
rapidly going into no-man's land especially if you are maintaining
the glide slope.
In the simulator we were unpleasantly surprised how quickly the
expanded localiser moved to off limits while at the same time we
were unable to detect a significant lateral displacement while
staring at the MAP mode even on the 10 mile scale.
At no stage did the expanded scale automatically revert to normal
scale even though we had reached beyond full scale displacement
on the HSI normal scale. I believe a safer design philosophy
would be to have automatic reversion from expanded scale on the
ADI to normal four dots if a significant lateral deviation occurred.

3rd June 2003, 00:04

bookworm
Join Date: Aug 2000
Location: UK
Posts: 2,949

#8 (permalink)

Thanks for the extra info Hudson and sorry to have confused: FSD
is Full Scale Deflection, but you gave the details I was looking for
anyway.

#9 (permalink)

5th June 2003, 06:24

M.Mouse

I find it hard to imagine why one would drift so far of the localiser
when using a flight director coupled to it.

Controversial, moi?
Join Date: Oct 2000
Location: UK
Posts: 1,231

The fact that flight directors can be so compelling has drawn


many people into following the FD indications despite the
instructions being in error.
What is the smallest scale on your MAP display?

5th June 2003, 10:58

None
Join Date: Jul 2000
Location: West
Posts: 364

#10 (permalink)

"I find it hard to imagine why one would drift so far of the
localiser when using a flight director coupled to it."
If it happens just one time, and being in raw data helps you catch
it......
It would be unlikely to see such an event in the typical intercept
scenario. But what about a busy approach controller trying to
squeeze you in (and you accept it), a busy cockpit (late
accomplishing checks), mountain environment, autopilot on,
approach gives you an intercept heading but the bank selector is
in 10 degrees instead of auto (you shoot through the LOC), it's
0800 but your body thinks it's 0200 (flew through the night), ....

5th June 2003, 15:45

seat 0A
Join Date: Apr 2002
Location: -11`
Posts: 177

#11 (permalink)

The simple answer is: just go around!


Once you are in the situation where you are looking at a full scale
deviation in the expanded mode, you are outside the safe
flightpath.
Even if you would have had the raw data display, you would still
be in the same place. In IMC there is only one option: go around!
Flying in the expanded mode also gives you the advantage of the
deviation warning.
Trying to recover from a position that far from the correct one, in
IMC is not a good decision. Just take your losses and try again
fresh.

5th June 2003, 23:08

Hudson
Posts: n/a

#12 (permalink)

SeatOA. Interesting re LLZ deviation warning. For some reason it


did not actuate in our 737-300 simulator. Must have another look
next time. Re go-around if you are one dot on expanded scale. At

this point the HSI will indicate less than half scale off centre line
which is no cause for a GA yet.
By the way the MAP is on 10 mile scale.

#13 (permalink)

6th June 2003, 10:18

dvt

I vote for MAP and raw data on the ADI.

Join Date: Apr 2002


Location: Lands End
Posts: 55

If you line up on final and your raw data will confirm/deny the
validity of your MAP display. If your MAP is good then you have
much more situation awareness. Ever had your ILS reciever fail
on you with no flags? It happens. I had it once. Ever had a false
capture, or a false GS lock on? How can you tell if you don't have
MAP mode up? Having the MAP mode up will save your situational
awareness and help you make better decisions.

#14 (permalink)

6th June 2003, 17:44

LEM
Join Date: May 2003
Location: The Roman
Empire
Posts: 801

Map display cannot be an official means to verify your LOC and


Glide Slope.
You must verify the LOC with an NDB or VOR radial, and Glide
Slope with a DME or LOM (or, as a last resort, an ATC radar
position).
Once you have done so, I agree a MAP display is better.
There can't be an absolute truth, the important thing is to make a
good valid check.
Display mode preferences depend on how your company believes
you are gonna tend to make these checks: if they think you will
be lulled into a false sense of security by a MAP display, they
might impose you the use of EXP VOR/ILS.
I would leave the choice to the captain, but ground him if he fails
to check his Glide Slope by the correct method.

#15 (permalink)

7th June 2003, 12:13

downwind

hudson,

Join Date: Aug 1998


Location: australia
Posts: 261

Is this a ETA course?, and is the sim VH-CZA (ex AN) or TJxx
(current QF 300/400 machine???????)
Also depends on the modelling on the sim computer!

8th June 2003, 21:11

gimpgimp

#16 (permalink)

Join Date: Jan 2002


Location: Melbourne
Australia
Posts: 32

Map shift and ILS approaches. Classic 737


All EFIS classic drivers flying into a remote strip with a u/s VOR
but serviceable ILS know the FMS updates its lateral position
once on the localiser. The map shift can be a few miles.
Once upon a time.. we had a false localiser capture a full three
miles off the centreline. I picked it up because
1. It happened sooner than I expected (after manoeuvring around
weather tru the localiser and back towards it expecting to become
visual at 3500 AGL.)
2. The track that was 212 instead of the required 222 and the
NDB showed us off track as well
3. On our request, Radar confirmed the three miles right of
centreline.
As we began to go raw data on the NDB to correct we became
visual, the FO corrected visually whilst I had a close look at the
EFIS.
Except for the giveaways above, all indications on the EFIS (on 10
mile scale) were that we were on centreline with the runway
ahead. Only a very close look showed the white track line on 212
missing the runway
I selected IRS 1 on the position page and entered, the map
adjusted, putting the runway where it actually was, to the left. As
I watched the map updated itself on the false localiser and put the
runway dead ahead again!.
As we approached the real centreline, the localiser needle
twitched, went full scale and then centred as we intercepted the
real centreline. At this stage the map runway was now off to the
right. A few seconds later the FMS updated itself on this new
localiser and put the map in front again.
I recount this little tale to explain why I always check all available
raw data before and whilst using the wonderful map mode.
If at any time the localiser pointer moves off the ADI expanded
scale I immediately select VOR/ILS on the EHSI.
I now fly both classic and the NG which of course has GPS and
map shift does not happen on the NG.

9th June 2003, 00:23

BOAC

#17 (permalink)
Quote:

Per Ardua ad Astraeus


Join Date: Mar 2000
Location: UK
Posts: 11,172

All EFIS classic


drivers flying
into a remote
strip with a u/s
VOR but

serviceable ILS
know the FMS
updates its
lateral position
once on the
localiser.
Sadly not QUITE
true, gg! Good ol'
BA appear to have
taken the feature
OUT of their
'BASpec' 737-400s
Turboprop Tips

Hi !
I have recently started flying a PT6A powered machine after thousands
of hours flying pistons.
I have been looking for a reference book that can fill me up with info on
the finer points of operating turboprops. Any suggestions ?? (Ive tried
Advanced Aircraft Systems, and Turbine pilots flight manual - neitehr
really spend much time on turboprops).
Some specific questions that pop to mind are:
1. Derated Vs Flat rated - What does it really mean ?? Whats the
operational difference to me ?
2. Starting problems - Hot, hung starts etc. Is batterty power the only
issue ??
3. Why am I told to turn the genny on the operating engine off when
starting the other ??
4. Should I be worried about autofeather failing in an engine failure
after TKOF scenario ?
5. The acft is certified under SFAR41 - what do I need to know about
this ?? It is considered a small aircraft in my country (under 5700kg) DOes SFAR41 have any significance ?
6. Best ways to detect water in the fuel ?? ******ed if I can tell if
theres water after a fuel drain. Is water as big a problem in JET fuel ?
7. What is the relationship between torque and Np ? Why does a change
in Np effect torque ?
8. Flight idle/Beta/ground range - What are they all about ?

Any other hints/tips or things i should be aware of ?? Common problems


(if any) with PT6A 's??
Many thanks !

29th August 2003, 13:21

Sheep Guts
Props are for boats!
Join Date: Oct 2000
Location: An Asian Hub
Age: 42
Posts: 875

#2 (permalink)

Is this a wind up?


If not what the
hell are you doing
flying one without
knowing some of
these pointers
allready.
1. Derated
ensures longer
engine and HSI
life., compared to
flat rated.
2.Hot or Hung
Starts usually
stem from 1 of 4
things
a. Battery Flat or
low
b. GPU or Battery
Cart flat or
incorrect
operation{ie. cuts
out after engaging
the starter}.
c. Starter
Generator just
about shot ie.
Brushes warn
beyond Limits
d. Any connection
in the system
thats lose or
arcing.
Now you notice
that most indeed
nearly all PT6s
have Starter
limitations which
are generally 25

seconds on 1
minute off, 25
seconds on 1
minute off, 25
seconds on 30-60
minutes off. This
depends on your
POH and type.
Also the start
sequence in your
POH usually has a
minmum limitation
of 12% N1or NG
or GG. This is only
a limitation and
should never be
attempted. My
rule of thumb on
this is if I dont get
16% N1 I wont
introduce fuel.
Anything above is
OK but watch the
temps carefully. If
you add fuel below
this and the
engine is allready
hot youll get a
breach of a start
limit TEMP, or at
least a reportable
temp to an
engineer. Your
POH will specify
this.
3.If you leave it on
you will draw a
large mount of
current through
the brushes in the
opposing
Starter/Generator.
Causing
execessive wear
and eventual
failure. When you
leats expect it.
Some Types
approve what thye
call Cross

Generator or
Generator assist
starts. Which are
usually for
emergencies only.
Also alot of
smaller Types alow
it B90 King Air and
some Early
Cheyenes I
believe. BUT
GENERALLY DONT
DO A CROSS GEN
OR ASSIST START
EVER.
4. No Just ensure
you identify and
confirm the
apropriate
Propellor Lever
and move it to the
Feather position
manually ASAP ,
after getting
control of the
Aircraft. Some
Aircraft will have
different speeds
for Auto Feather/
nil Auto Feather
operations beware
of this.
5.SFAR41 dont
know this will
research it
6.Well the best
way is to let it
settle over night.
Water is easily
suspended in Jet
A1 compared to
Avgas. That is to
say more
"Hydroscopic" I
believe the correct
term is. Then
drain using a
standard strainer.

7. Same way Prop


RPM effects MAP
in a RECIP REALLY
8. Get out your
POH and get
someone to give
you a ground
school Cause I
havent enough
time here.
Ofcourse this
depends on what
Type or - dash
version of the PT6
you operate and
Aircraft .
Regards
'Sheep

#3 (permalink)

29th August 2003, 15:05

cjam
Join Date: Jul 2002
Location: australia
Posts: 365

Topper, one of the issues with water in the fuel is that microbes,
little bugs called cladosporium resinae , can use the oxygen in the
water in conjunction with the carbon in the jet-a to live happy
little lives in the fuel system procreating willy-nilly resulting in a
sludge that can, in time, block filters etc. If there is no water, then
there's no clad resinae.
If the water is not held in suspension it will sit at the bottom and
is easy to see,most of the time it is in suspension and the only
real way to check properly is with a water test kit though.
That is all for now....you can take a fifteen minute break.

#4 (permalink)

29th August 2003, 22:54

Sheep Guts
Props are for boats!
Join Date: Oct 2000
Location: An Asian Hub
Age: 42
Posts: 875

Cjam spot on chap. Forgot that. Also there is treatment for this
stuff its called BIOBOR. You add it to your tank after filling. Its
suppose to eliminate the bug.
Regards Sheepster
P.S. TIME FOR LUNCH, I here theres a good Wendys down the
road.

29th August 2003, 23:11

john_tullamarine
Moderator
Join Date: Apr 2001
Location: various places .....
Posts: 3,768

#5 (permalink)

I don't fly PT6 but have had some peripheral involvement with
them so the following has more of an engineering than pilot slant.
1. Flat rating. Normal engine characteristic is for output to
increase as OAT decreases at constant altitude. On many engines
the mechanicals become the limiting factor (typically gearboxes,
combustor pressures, or whatever) and the machine output has to
be restricted at OAT below that at which the mechanical limitation
becomes critical. Alternatively, the OEM may chose to limit the
engine to a lower than maximum output for a variety of reasons.
In this lower OAT region, the engine is said to be flat rated as the
power or thrust output is limited to provide the relevant maximum
which the mechanicals can tolerate or the OEM choses to adopt.
Operationally this means that the RTOW fall off with increasing
OAT for a given runway is modest until the engine is no longer flat
rated and then falls off much more rapidly with increasing OAT.
Derated certification. If an engine is capable of a certain output
with whatever level of progressive deterioration in service, then it
is reasonable to presume that the mechanicals and the hot end
will last longer (ie cheaper maintenance) if we hold the output
below the maximum. There are two ways to approach this
(a) flexible thrust settings (normally used on jets) where the
engine is operated in a manner which uses a lower than maximum
thrust output .. but the limitations are based on the certificated
maximum thrust capability. For US rules, the maximum reduction
is 25 percent.
(b) derate thrust where the engine is limited by the electronics or
mechanical settings to be able to produce no more than a specific
percentage of the maximum capability. This has significance cost
implications and may have some performance advantages in
certain circumstances. Many of the modern jet engines are
certificated to several thrust rating levels, selectable by the
flightcrew via FMS or similar wizardry.
Derate and flex can be used in conjunction. This may well result in
considerably lower thrust output than the 25 percent flex limit for
the maximum thrust certification.
2. Cranking power probably is the most significant factor. Others
include wind (especially if from astern).
3. Cranking loads during turbine start are very high .. typically
something in excess of 1000 amps. The generator is not much
use for these sorts of current drain figures. The secret is to use
ground power for starts to save the ship's battery

4 Failure of autofeather is very serious on a direct turboprop


(typically the older engines) as the windmilling prop has to drive a
whole bunch of turbomachinery through the gearbox. Much less of
a problem on a free turbine but still of interest.
5 SFAR41, now defunct but still relevant to older SFAR41 aircraft,
upped the ante of FAR23 a bit towards FAR25 to facilitate the
development of early commuter aircraft. There is a useful FAA
memo on the subject which you might like to read.
6 Two significant problems with Avtur is that the fuel's colour
often masks water and water may stay in suspension for
considerable time. As a result it is usual to employ chemical
means to detect the presence of water and there are several
processes in common use.
7 I will dig out some charts for a couple of dash variants and
come back with some numbers ..
8 Better for rated pilots to address this one for the particular
engine.

#6 (permalink)

29th August 2003, 23:16

Sheep Guts
Props are for boats!
Join Date: Oct 2000
Location: An Asian Hub
Age: 42
Posts: 875

There are not many common problems with the PT6. The early
models had problems with low TBOs and HSI intervals. But as the
engine has eveloved over the years the Modern Machines are
much better. Having TBOS of around 3500 Normally with
exensions upto 5000 hrs for FAR 135 and 121 ops ( Charter and
RPT), THE ENGINE HAS A GREAT RECORD! But dont belive its
bullet proof cause it aint. I have around 2000 hrs on PT6 powered
Aircraft without a problem,( except a machine I wa sscheduled to
fly had a compressor failure on the ground after start ). Another
mate of mine had one go in the Air, but hed had 3000 hrs on PT6s
at that time. Both these failures were incidentally on PT6-20
Engines which have had problems in the past with low TBOS and
single stage nozzles which give high Start temps. The later
versions and had dual stage nozzles which gave a staggered start
temp much better.
See you tommorow
Regards
Sheep
P.S. Your home work tonight. Go and get that POH and read about
the PT6. Define for me the BETA Range and Reverse range of your
props in degress and the specified limits on N1 for these
operations and temp limits too?

#7 (permalink)

29th August 2003, 23:34

604guy

Topper,

Join Date: Jun 2003


Location: Canada
Age: 55
Posts: 195

Just have a few seconds here so wont go into a lot of depth for
answers. First off, I ASSUME that you are going to receive some
formal trainingRIGHT?
Hot starts Keep in mind that about of the air going through
your engine is for cooling purposes, only the remaining is used
for combustion. The amount of air being drawn through the
engine is dependent on the speed of the engine (revolutions of
the gas generator). The slower the engine the less air is being
drawn in. The non-variable is of course the amount of fuel being
introduced at engine start. So if your engine is not turning fast
enough when you introduce the fuel you can have a hot start.
Reasons can include low battery/inadequate GPU voltage. (Neither
of which should happen because the pilot is supposed to check
that before attempting a start). Mechanical problems with the
starter/generator etc can also be a cause but it is the pilot that
decides whether to introduce the fuel or not.
Xgen starts dependent on the aircraft type. Some applications
call for xgen start as normal procedure but this of course would
be fully covered in your formal training.
Water in jet fuel in addition to previous comments remember
that water goes into suspension very readily with jet fuel.
Attempting to drain fuel after the aircraft has been towed etc is a
waste of time. The time to do this is after the a/c has been
motionless for several hours.

#8 (permalink)

29th August 2003, 23:36

flyer75
Join Date: Aug 2003
Location: worldwide
Posts: 118

Airplanes...sorry "AIRCRAFTS" technicalities

First day on PPRuNe...


I have read about the poor guy who had the mind to ask technical
questions about the engines hes using to defy his own
gravity...scary...
On the other hand I spent 4 to 5 years flying metro3s and be1900
single pilot for a freight operation in South Dakota,USA and the
only thing i needed to know was to fly an ils,ndbs whats that
again?avoid cbs as much as i could in KANSAS and make sure no
red light even thought of lighting up during start...
A part from that what else does one need to know...checklist and
memory items/flows are there for you...they are your

friends,,eventhough checklists can get old and dirty after a few


freight dogs used them to scratch the ice off the leading edges
and props.
Sure enough I decided that the red eye life was getting strainious
so decided to convert my FAA ATPL o the glorious JAA ATPL.
Needed ground school so here i went to OXFORD..where they
filled my poor freight dog head with technical information during
eight months..first pass in all subjects..i must be technically fit
and somehow clever..did help a bit to pass my IR/type rating with
the CAA.lEARNING THE 737 MANUAL IN A 10 DAYS COURSE IS A
BIT TRICKY BUT HEY THANX OXFORD TO HAVE woken up my
memory..i passed..
Well a year later i could barely tell you how to calculate Mach
number from TAS or explain how a piston engine works or what a
great circle/"RHUM" Line is......
Bottom line....experience is the main factor in learning..if its
getting too dark upfront and no ones on the frequency..go back to
where you come from....
Just dont burn out your engines trying to learn their limits by
experience..a little reading is essential..
Tailwinds,
Flyer 75

#9 (permalink)

30th August 2003, 06:54

IHL

TopperHarley:

Join Date: Feb 2001


Location: Canada
Posts: 440

Cross generator starts on PT6 powered KingAirs can cause the


operating engine to exceed ITT limits. Our company procedure
was to have the operating engine at High idle (70%) before
attempting a start, and monitor the ITT of both engines during
the start. Even at hi idle I've noticed a 50 Degree rise in ITT on
the operating engine.
NB: you got to be thick skinned to post on Tech Log.

30th August 2003, 07:28

604guy
Join Date: Jun 2003
Location: Canada
Age: 55

#10 (permalink)

IHL
The start checklist for many of the newer Kingairs requires that
the operating engine be in high idle before starting the xgen start
procedure. I can assure you that it does not cause any ITT

Posts: 195

problems whatsoever.

#11 (permalink)

30th August 2003, 09:41

Sheep Guts
Props are for boats!
Join Date: Oct 2000
Location: An Asian Hub
Age: 42
Posts: 875

Ok Topper,
Please dont keep us in suspense any longer what Type of Aircraft
and PT6A version is fitted? Lets see, it could be one of the
following:
1. B200 KingAir-----------PT6A-42/41s
2. B90 King Air------------PT6A-20/21
3. DHC-6-300-------------PT6A-27
4. DHC-6-200-------------PT6A-20
5. EMB110-----------------PT6A-34
6. DHC-7-------------------PT6A-50 BY 4
7. B300 KingAir-----------PT6A-60
8. B350 King Air----------PT6A-60
9. Shorts 360-------------PT6A-67
10.B1900------------------PT6A-67
11.RHIEMS406------------PT6A-112
12.B100 King Air----------PT6A-28
13. CaravanC208---------PT6A-114

Thats all I know. Not sure on Shorts 330, Cheyenes,


Y12(CHINESE JOBY), are there any others?
Sheep
Last edited by Sheep Guts : 1st September 2003 at 10:37.

#12 (permalink)

30th August 2003, 14:53

TopperHarley

Thanks for the info guys.

Join Date: Aug 2003


Location: Aust
Posts: 123

For better or for worse, the ground course I did placed a lot of
emphasis on rote learning numbers. I have nearly every number
in the POH comitted to memory (What temp does the CABIN AIR
DUCT O'HEAT anunciator come on at ?) but perhaps it was
assumed that I knew the fundamentals.
A very experienced pilot once told me "You dont know what you
dont know" - I think I am in that position.
Sheepguts - Its the -112 in the F406
WRT my homework - You wouldnt believe this sir but you see I
was looking after a friends dog.... I only went out for five

minutes..... !
The power leaver pedestal is marked in three ranges. Flight,
Ground Idle, reverse. With a gate between flight and ground idle.
Very little reference to Beta in the POH.
One mention in the definitions part of section 1 "BETA MODE Engine operational mode in which propeller blade pitch is
controlled by the cockpit power lever. Ground ops only"
Then later on that "To acomplish reverse pitch, the power levers
are retarded beyond idle, over the gate and into the BETA
position"
From the limitations section of the POH, prop blade angle at 30"
station:
* Feathered = 85.5 degrees
* Low pitch = 18.5 degrees
* Full reverse = -13.5 degrees
and engine limits:
* Idle = 685ITT
* Max reverse = 1382 ft-lbs, 725ITT, 101.6%Ng
Was that what you were after sheepguts ??
Regarding Flat/De rating - A lot of people have told me that this
engine is DErated, however the POH says that its flat rated to
500SHP. Ive also heard the phrase "Bulletproof" mentioned in
close proximity to "derated" (should I be worried??). Is there a
laymans definition of derated that is perhaps confusing me ?
Thanks again for your help people - If youve got any more useful
comments please keep them coming - Im keen to find out all the
things I dont know !

1st September 2003, 02:47

#13 (permalink)

hptaccv
Join Date: Jan 2001
Location: EU
Age: 32
Posts: 50

'flight idle' is a few % higher than 'ground idle', basically a


different power setting.
'Beta Range' can be translated as reverse, or negative blade pitch.
This is controlled by the power lever (although it is basically a
blade pitch change) in order to be sure of the engine being in idle
before reversing.
I'ts been a while, but i also remember the condition lever having
two detents, high and low idle, basically only used for crossengine starting in hot&high conditions. PT6-61 (cheyenne pa42)

regarding flat- and derated engines:


relative to outside air temperature:
flat rated meaning the engine is rated to a certain power limit,
before reaching itt limits for example. On reaching an ambient
temperature which leads to egt limitation, this will be the limiting
factor, thus leading to less power available.
a de-rated engine on the other hand, is limited to an itt of say
700c, even though the manufacturer limit might be 750c. This
reduces maximum available power throughout the whole
temperature range.
I stand to be corrected! (excuse my stumbling english!)

#14 (permalink)

1st September 2003, 05:07

redsnail

Great thread. Never too old to learn stuff.

PPRuNe Handmaiden

Sheepy,
Shorts 330 = PT6 45
Shorts 360-100 = PT6 65R
Shorts 360-200 = PT6 65AR
Shorts 360-300 = PT6 67. What letter is after it I don't know.

Join Date: Feb 1997


Location: Duit On Mon Dei
Posts: 3,374

#15 (permalink)

1st September 2003, 16:41

F111

-67R on the Shorts 360-300.

Join Date: Jun 1999


Location: QLD, Australia
Posts: 177

#16 (permalink)

1st September 2003, 16:48

TopperHarley

Thanks again for the info guys...... It is really appricated.

Join Date: Aug 2003


Location: Aust
Posts: 123

Has anyone got any general hints or tips that might make teh
transition easier or safer ??
Any checkies that see people do things in turboprops that make
you cringe ??
Thanks again

1st September 2003, 16:59

Crossunder

#17
(permalink)

Well, let's
see...

Aviator
Join Date: May 2001
Location: Norveg
Posts: 450

Auto Take Off

Hi

Your
Company
will
(hopefully)
introduce
you to
their SOP,
and you
will also
receive a
technical
course
prior to
flying
training.
That
should
clear up
any
questions
you might
have.
Apart from
that; flying
turbo
props is
the easiest
thing in the
world,
except
flying jets.
No tricks,
no funny
business,
just fuel on
or fuel off.
Never
understood
what the
fuzz is
about.

I tried searching the web for information on two topics that have been puzzling
me for a long time (but had no success, maybe you guys can help).
1) Does any Commercial Airliner have an auto takeoff facility? Somebody
mentioned the L1011 to me once (it wasn't the guv either!).
If not, why not?
2) I read a book once, perhaps it was "handling the big jets", however I recall
the published date as 1974.
This book had some pics inside the cockpit of a BOAC 747-100/200, and in
between the pilots just in front of the throttles was a very large vernier type
dial named "SLEW" - what did/does this do???

#2
(permalink)

3rd September 2003, 23:41

Golden Rivet

Not that I
know of most
autopilots
can only be
engaged
after a
specified
altitude
has been
attained
( eg 400
ft ).

Join Date: Feb 2001


Location: The grim north....
Posts: 343

As always
I'm happy
to be
corrected.
GR

3rd September 2003,


23:44

fatboy slim
Join Date: Nov 1999
Location: UK
Posts: 154

#3 (permalink)

2) I think SLEW would be Yaw Trim in today's money.

4th September 2003,


11:55

QAVION
Posts: n/a

#4 (permalink)

"Not that I know of - most autopilots can only be engaged after a


specified altitude has been attained ( eg 400 ft )."
Depends on the mode, GR, and, of course, the aircraft.
I have had no problem engaging HDG HLD, HDG SEL, V/S and
FLCH on a 747-400 on the ground. Theoretically, you could use
FLCH for takeoff, if it wasn't so risky doing so (I believe someone
has already tried this, anyway, in real life and got away with it).
Yaw control would be another matter. It's not engaged until the
aircraft is in the air (plus or minus a few seconds).
Autotakeoff is achievable, technologically, but it carries a lot of
risks. You would probably need a computer like Deep Thought to
consider all the things that could go wrong and then take
appropriate action... something which the pilots have already
mentally prepared themselves for before they push the TOGA
button.
Regards.
Q.

4th September 2003,


19:49

AhhhVC813
Join Date: Sep 1999
Location: U.K
Posts: 67

#5 (permalink)

I believe Airbus were looking to introduce an auto take-off mode


on the 340-500/600, but couldn't get the agreement of the
airlines who had ordered them.

5th September 2003,


06:14

Cornish Jack
Join Date: Feb 2000
Location: UK
Posts: 737

#6 (permalink)

trium 16
I think that the L1011 reference was partly correct. If memory
serves me correctly (not very often, nowadays
) Lockheed
included the ability to take-off with Control Wheel Steering
engaged. This was a limited autopilot function which allowed
manual input to override the A/P stabilising function in pitch and
roll and allowed altitude capture.
I don't know of any operator whose OM allowed this function to be
used for take-off.
As in most areas, the L1011 was light years ahead of its
contemporaries.

7th September 2003,


14:41

#7 (permalink)

*Lancer*

I heard they're looking at it quite seriously for the A380...

Join Date: Mar 2000


Location: Australia
Posts: 586

It's not that hard to do - apparently Boeing have already admitted


it's only a minor modification to the autoflight suite to get it going
(hell, enough drones do it already). Auto-Rotate pitch mode, and
Rollout (or similar) isn't that complicated - even with an engine
out. QAVION, there are a lot of considerations, but like landing,
even if you get a No-Autoland you can still take manual control
without significant consequence.
The alternative is if the automatic feature simply just controls the
elevator to avoid scraping the tail...
I really hope the insurance companies realise the overwhelming
negatives of mandating auto take-off and landing!
Lancer

8th September 2003,


02:06

#8 (permalink)

Larry in TN

What's the point?

Join Date: Jul 2000


Location: USA
Posts: 32

Autoland makes sense because it allows you to land in lower


visabilities. What would auto takeoff give us that would justify the
expense of the equipment, training and maintanance?

8th September 2003,


05:35

B73567AMT
Join Date: Aug 2003
Location: New York
Posts: 69

8th September 2003,


11:04

QAVION
Posts: n/a

#9 (permalink)

Software in the FCCs on the B737 and B757 prevent Autopilot


mode engagement for takeoff. Yes Hdg Sel and other modes can
be selected on the gnd, but they are not available once the A/P is
put into Takeoff mode (TOGA). The modes are inhibited until 400ft
AGL.
Go ahead try it, it can't be done.

#10 (permalink)

"Yes Hdg Sel and other modes can be selected on the gnd, but
they are not available once the A/P is put into Takeoff mode
(TOGA). The modes are inhibited until 400ft AGL."

Not sure I understand this, B73567AMT.


On the 737, if you have, say, Level Change and HDG SEL engaged
(CMD) on the ground, will the TOGA switches do anything? I
assume these modes would override the TOGA arm modes(?)
Also, on the 757 with, say, HDG SEL/FLCH engaged on the ground
(if possible), I also assume TO/TO won't be annunciated. Would
pushing the N1/EPR button change change the FMA
annunciations? If so, could you bypass this by pushing the thrust
levers forward manually?
Thanks.
Regards.
Q.

8th September 2003,


15:56

B73567AMT
Join Date: Aug 2003
Location: New York
Posts: 69

#11 (permalink)

I'll backoff the 757 until I can find my material...but on the 737, it
is written in the software that TOGA will cause the A/P to
disengage.
I have to find it in the algorithms, but I know it is there for a fact.

11th September 2003,


16:55

OneDotLow
Join Date: Jul 2000
Location: Australia
Posts: 129

#12 (permalink)

With respect to autoland.... the AFDS uses the LOC signal to align
the aircraft with the runway, then takes a snapshot of the ground
track just prior to annunciating rollout as the active mode.
My question is... how would the aircraft know what track to
manouver the a/c on for a takeoff.
GPS? Maybe, but I dont think that this can be relied on to give an
accurate enough position.
Having said all of that, maybe in the computer control laws (like
the 777 has), there could be someting added to minimise
tailstrikes... just a thought.

12th September 2003,


01:31

#13 (permalink)

NigelOnDraft

As alluded to above - WHY?

Join Date: Jan 2001


Location: UK

Autoland (with close pilot supervision) was only designed and


implemented to get around a weather problem... i..e for

Posts: 1,331

commercial reasons.
I see no commercial advantage to "auto-takeoff"...
At a practical level, probably far easier than autoland to
implement... but if it requires the same level of pilot supervision
as autoland, why not just get the pilot to do it?
NoD

14th September 2003,


09:04

Zagor
Join Date: Sep 2003
Location: hong kong
Posts: 3

#14 (permalink)
auto take off

I see potential hazard like tire temperature,long taxies brake


temeratures that could effect the T/O performance, the main
reason why is not necessary to improuve safety by automation.
however is uncleas to me why in autoland there are no
implementation of the Flap and gear extension performed
according to FMC speed.
It will bring safely down as plane with total incapacited crew.

14th September 2003,


13:04

#15 (permalink)

Ignition Override
Join Date: Jul 2000
Location: Down south, USA.
Posts: 1,599

To add to what Nigel O.D. and others said, more automation


during the most critical phases of flight does not mean that more
safety will result, and probably much less. It often
disappoints/dismays me to read that more layers of technology
will always be pursued if tiny savings in operating costs can (in
theory) be achieved. Our upper mgmt would let us fly airliners
solo if the FAA and insurance companies could find it legally and
financially feasible.
The Airbus (begun with the fly-by-wire 320 series) design
philosophy seems to put much less faith in pilots (or their
experience levels with some operations?) than Boeing's
automation philosophy. Many of our pilots have flown both types
of products.
Do most aircraft design engineers, in a general sense, look down
on human pilots? If so, then it is doubtful whether very many of
them have much IFR experience in a real airplane. A concept is
one thing, but experience and some judgement can not

necessarily be digitized, prioritized and programmed, can it?


Last edited by Ignition Override : 14th September 2003 at 13:18.

14th September 2003,


15:05

#16 (permalink)

NigelOnDraft

Zagor

Join Date: Jan 2001


Location: UK
Posts: 1,331

<<It will bring safely down as plane with total incapacited


crew.>>
I'm afraid you are completely missing the point. You seem to
think Autoland is controlled from the ground, and/or the pilots set
it all up in the cruise or something.
All Autoland does, and all it is designed for, is provide a
mechanism to get around the problem of poor visibility in the last
200'.
If you are looking for a solution to the "incapacitated crew" issue,
you are opening up a totally new can of worms... and lowering
Flaps and Gear is the most minor of the concerns that would need
to be addressed.
Even on the Airbus, an Autoland, safely conducted, involves
extensive crew interaction and systems monitoring. There is even
a half page of "deficiencies" that limit or prevent Autoland that are
not monitored by the aircraft systems...
NoD

#17 (permalink)

14th September 2003, 19:02

PAXboy

Quote:

Paxing All Over The World


Join Date: May 2001
Location: Hertfordshire, UK.
Posts: 4,628

It often
disappoints/dismays
me to read that
more layers of
technology will
always be pursued if
tiny savings in
operating costs can
(in theory) be
achieved.
Having worked in
technology for 23 years
- I heartily agree!

Bean counters will


always opt for fixed
costs over flexible i.e.
equipment against
humans. They can plan
for fixed costs with
purchase, depreciation
and (so a fair degree)
maintenance, across ten
years. Humans make
unplanned demands and
may resign, go on strike
or experience long term
illness, needing contract
(expensive)
replacements. Worst
case, ccxd sectors. With
a box, you go and buy
another one and if the
maintenance costs rise,
you change the
maintenance supplier.
Ensuring, if at all
possible, that if the new
lower cost maintainer
makes a mistake, that
he will pick up the
liability of a claim.
Last week, I was in a
meeting of a very senior
and experienced
engineer (not aviation)
and he said,
Quote:

The accountants
optimised the hell
out of everything
and then they got
bored. So they
decided to run the
companies as well.
Now we're all in
trouble.
Min visibility for departure in LVO's

I wonder if you can share some information on the minimum


required departure visibility in Low vis operations.

Recently our CAT II has been downgraded to CAT I due to LOC


interference and I was just wondering if that would chnage the min
vis requirement for departing aircrfat from the airport in low
visibility conditions. In a CAT II situation, the min required vis was
125 m and if the TD RVR fails, then it increases to 150 m and both
mid point and stop end RVRs must be serviceable for a rejected
take-off. Now, in a CAT I situation, the normal RVR for landing is
550m and I must assume that in order to depart safely, the min
departure visibility must be 550 m in the event that the crew has to
return for an immediate landing after take-off in an emergency. If
the dep. vis is less than 550m and you are allowed to depart, does
it require a take-off alternate which must be within a short
distance/time away?
Do you expect controllers to protect the ILS sensitive area when the
vis drops below 600 m and 200feet? Our CAT I holding points are
within the sensitive areas and the question is whether it is
permisable to hold aircraft at the CAT I holds when there is arriving
aircraft on the approach? Using the CAT II holds will obviously take
care of the situation, but it requires additional spacing on final
between arrivals in order to get a departure away. You are probably
looking at a min 15 nm in trail distance between arrivals if a
departure is inserted in between arrivals.
Any information on low vis ops would be appreciated.
Thanks
Fart

#2 (permalink)

17th October 2002, 19:37

Pilot Pete

fart

Join Date: Aug 2000


Location: EGCC
Posts: 1,708

Quite right in your


assumptions. If you
need to make an LVO
approach you need to
check with ATC that
they have LVO's in
force to ensure
protection etc. So it
follows that you would
not want to be making
an autoland in LVO
conditions if ATC were
still using the CAT I
holding points.
You are also correct in

thinking that the


spacing is increased
significantly in LVO
conditions (any
ATCO's got the exact
gen?) and delays are
to be expected so
make sure you have
plenty of fuel for going
round in circles at
your destination
should the forecast be
bad. It's also worth
considering your
strategy when you get
to the overhead and
enter the hold. ATC
will generally give you
the expected delay in
the hold, if that is
going to put you close
to a diversion decision
then it may be worth
diverting early, getting
into your alternate
and re-fuelling with
plenty (golden rule;
don't tech stop and
end up diverting again
because you didn't put
enough on!) of fuel
then race back to your
original destination
which is now less busy
because everyone else
has now diverted to
where you've just
been! Happened to us
once going into
Manchester early
morning with no low
vis forecast but LVP's
in operation when we
picked up the ATIS.
Every man and his
dog entering the stack
(all with the same
forecast as us so noone had much fuel to
be playing with) and
we diverted before
even reaching Dayne,

refuelled at Brum as
everyone else followed
us in, departed and
landed at Manch
probably 30-45 mins
before the next one!
125m is our absolute
minimum as you point
out the same rules
apply for RVR reports.
The reason for a
departure alternate
within a certain
distance (I've flown
with a couple of outfits
now and one used
400nm and the other
250nm) is not down to
ILS category primarily,
more to do with worst
case, i.e. engine
failure on T/O which
precludes an autoland
(on the 757 you can
only autoland with one
engine if you have
armed 'approach' prior
to losing the engine),
but that does lead to
some debate on is it
wiser to fly possibly a
considerable distance
to an alternate on one
engine when you have
a perfectly serviceable
airfield in LVP's nearby
(don't shoot me down
I didn't say I'd do it,
just there is a debate
about it!). So even if
the ILS was CAT III
we couldn't make an
approach in the EFATO
case.
Hope that
helps......there's
tonnes more for a
winters day reading!
PP

#3 (permalink)

17th October 2002, 19:59

fart
Join Date: Jun 2000
Location: Somewhere you
cannot find me!!!
Posts: 7

Thanks

Pilot Pete,
Thank you for the info - it is indeed very helpful.
Regarding the spacing on final, and by the way, I am not from or
working in the UK, our spacing criteria is as follows: (Somewhere
in ME)
30 nm in trail between arrivals if there is a departure in between.
20 nm in trail with no departures.
Parallel runway ops suspended.
Arriving aircraft must be established (vectored) outside 10 nm
from TD at a speed not above 210 knots.
Departing aircraft must have crossed the LOC antenae at the
upwind threshold before the arrival reaches 10 nm from TD.
Approach ban policy - need 350 m RVR at 1000 feet for CAT II
approach.
Runway incursion alarms active (GMR/SMR)
etc.
cheers
Fart

#4 (permalink)

19th October 2002, 18:55

GlueBall
Join Date: Jun 2001
Location: UTC +8
Posts: 2,124

I think that you are mixing apples and oranges. Departure


minimums have nothing to do with landing minimums. The only
criterea when departing from an airport that is below landing
minimums is to include a "Departure Alternate" for planning
purposes. Unless an airport operator specifies higher departure
minimums, our Ops Specs allow us to depart with 600 (feet) RVR,
provided that runway centerline lights are operative and that at
least two out of three RVR detectors are operative. Our Jeppesen
airport charts include FAR 121 takeoff minimums for each runway.
Last edited by GlueBall : 19th October 2002 at 19:01.

#5 (permalink)

21st October 2002, 00:23

Pilot Pete

Glue ball

Join Date: Aug 2000


Location: EGCC
Posts: 1,708

See what you are saying and departure minima have nothing to do
with landing minima, but you say

Quote:

The only criterea when departing from an airport that is below


landing minimums is to include a "Departure Alternate" for
planning purposes.
which is not strictly true as with a minimum departure vis of 125m
RVR and a CAT IIIb approach minima of 75m the airfield landing
minima could be less than the departure minima, thus the criterion
for the departure alternate is more about conditions precluding a
return to that field i.e. worst case including technical problems such
as single engine, as mentioned above on one engine we could not
do an autoland if the engine failed before arming the approach
mode. So no, minima for approach and departure have no
comparison, but if you are going to depart you need to know where
you can go if you should not be able to return.
PP
edited due too much cabernet sauvignon
#6 (permalink)

21st October 2002, 21:52

GlueBall
Join Date: Jun 2001
Location: UTC +8
Posts: 2,124

Our FAA Ops Specs criteria for a "Departure Alternate" is that the
airport is within 2 hours cruise (still air) with one engine inop. If
that condition cannot be met, then obviously takeoff is not
permitted...until an airport within two hours cruise in still air
(including the airport of origin) meets alternate landing weather
minimums.
I think what you need to keep in mind is that a minimum visibility
takeoff is not predicated on being able to immediatley return to
the point of departure. I have departed many airports many times
in weather conditions that precluded a return in case of an engine
failure.

#7 (permalink)

21st October 2002, 23:19

BlueEagle

PVD (Para Visual Display).

Join Date: May 2002


Location: Australia
Posts: 2,252

On the B747-400 I flew we had a PVD display, (a black and white


barbers pole), on the coaming, centrally in front of each pilot.
The PVD requires a useable ILS localiser signal that has to be
selected prior to take off. When the aircraft is on and pointing
down the runway centreline the display will be stationary.
During the take-off roll, the PVD will provide centreline guidance
when the aeroplane exceeds 2.5 degrees or greater deviation
from the runway centreline. The display moving to the left is a
command to turn to the left and vice versa, the faster the display

moves the larger is the command.


This equipment allowed to take off when the RVR was down to,
but not, below 100 meters.

#8 (permalink)

22nd October 2002, 10:47

Pilot Pete

Interesting Blue

Join Date: Aug 2000


Location: EGCC
Posts: 1,708

Was the slant view from a 747 cockpit worse than for a smaller
type ie. 737? I mean was it possible that a 737 pilot would be
able to count half a dozen centreline lights whereas you would be
able to see less in the same conditions? Was this a reason for the
PVD or was that just to get your minima down to 100m? What
was minima for T/O without the PVD?
PP

22nd October 2002, 23:06

BlueEagle
Join Date: May 2002
Location: Australia
Posts: 2,252

24th October 2002, 15:48

m&v
Join Date: Dec 1999
Location: delta.bc.canada
Posts: 259

24th October 2002, 23:08

#9 (permalink)

View from cockpit of 747. It was not as good as from a 737, for
sure. Our landing minima DH for Catlll was 20' whereas the
757/767 would be 0'.
Reason for PVD. Not all operators have the PVD fitted but I am
guessing that it was originally intended to get minima down to
RVR 100' as the T/O minima otherwise would be as published in
the Jeppeson.

#10 (permalink)

Pilot pete,yes you will get the scenario whereas the 'landing'vis is
lower than the T/o vis-due to the theory that the Fed's give more
leeway to a Dynamic situation than the static situ (on the ground)
One needs a certain RVR for T/o,as stipulated by the Ops spec.
namely 600'-1200'feet (north america)depending on the
airport/runway lighting and equipement.If the weather is on
Minimums for T/o-one needs a T/o alt(within 1 hour twins/2hours
more than twin)...That having been said ,one can always return in
the event of EMER'on a CAT3 arrival(Sim practice heart attack
etc).If the CAT3 goes bust,then the Cat 2,or the Cat1,with the
required increase in RVR,is required for the return.
Basically 600 rvr for T/o needs Centre line lights/markings-if no
centre line-1200 applies

#11 (permalink)

Pilot Pete

Blue

Join Date: Aug 2000


Location: EGCC
Posts: 1,708

Thanks for the reply, interesting reading.


m&v
I think we are saying the same thing in a roundabout way. Here in
the UK with my Ops Manual we can depart down to 125m RVR
with certain a/c, crew and airfield requirements met. If the airport
is CATIIIb operational then we could undoubtably make an
approach to minima if we required to return to the field using the
published CATIIIb minima (usually 0' or 14' DH and 75m RVR),
but only, and this is the point I was trying to make, if the a/c,
crew and airfield were to meet the operational minimal
requirements. It therefore follows that the T/O alternate has
nothing to do with the departure minima but is there when a
return to this airfield is precluded due to downgrade of equipment
meaning that the revised minima would not get us back in; the
example I used was in the EFATO scenario where we could not
make an approach to autoland if the engine had failed before
arming the APPR mode, therefore a suitable departure alternate is
required within a certain distance/ flight time with weather
conditions forecast as good as or better than the required minima
dependant on the approach aid to be used at that field.
Phew, hope that clears my feelings up on this matter!
PP

#12 (permalink)

25th October 2002, 06:51

Sick Squid
Moderator
Join Date: Apr 1998
Location: Exile on Guildford
High Street
Age: 44
Posts: 1,259

...another little issue to consider... there should be a limit


somewhere in your ops manual for take-off below which LVP's
must be in force... 400m for my company.
To go down to our absolute minimum (150m/125m) requires
protection.. quite why when the PVD is not required for that I
don't fully understand (apart from the in-trail landing/departing
aircraft consideration) but there you go.
With planned Cat 3a single engine capability, you really have to go
some to lose the return option close to Take-off minima at most
major fields. But as others stated better than I, they are two
completely seperate calculations.
Sick Squid
Last edited by Sick Squid : 25th October 2002 at 20:37.

#13 (permalink)

25th October 2002, 22:39

m&v
Join Date: Dec 1999
Location: delta.bc.canada
Posts: 259

Agreed PP
The whole concept of the Dept Alt is becoming deemphasised
these days with the autoland/eng out capability of the fleet
types.The only case is total power failure at the Dept field(aux
should kick in),thunder storms,or strong winds etc.
The concept seem's to be an 'olde' holdover from the Cat1 days.
Cheers

#14 (permalink)

26th October 2002, 10:48

Pilot Pete

Sick Squid

Join Date: Aug 2000


Location: EGCC
Posts: 1,708

Got me looking for the RVR limit below which LVP's would have to
be in force for departure but could not find one specified
anywhere. I would therefore assume that if I were to depart with
less than CAT I RVR approach minima then LVP's should be in
force?
You also mention CAT IIIa minima allowing return on one engine.
Does your type allow for a planned autoland in the case of an
EFATO? That I find very interesting as the 757 does not allow for
this.
Could your required 'protection' when down to absolute departure
minima be anything to do with ensuring you line up on the
centreline by reference to the localiser?
PP

#15 (permalink)

27th October 2002, 09:28

Spitoon
Join Date: Apr 2002
Location: Europe
Posts: 2,066

As a controller I can't really comment on Ops Manual procedures


but bear in mind that here in the UK LVPs can involve far more
than just using CAT III holds. Many other restrictions are
introduced to limit the number of vehicle (and sometimes aircraft)
in airside areas and often to physically block runway access points
that will not be used. At airports that do not have surface
movement radar it is common for movement on the
manoeuvering area to be limited to one at a time. These
procedures are intended to prevent unauthorised runway
incursions but may also be taken into account by the Ops Manual.

#16 (permalink)

30th October 2002, 07:11

m&v

Yes'

Join Date: Dec 1999


Location: delta.bc.canada
Posts: 259

the A320's good for an auto land on one engine,although for a


long time 'local' fed's disallowed it.In Canada still different
'approvals' for different Co's.
In Catt3 ops the engine has to be 'secured'and checks complete
at least 1000' agl.....

1st November 2002, 16:49

BusBoy
Join Date: Apr 1999
Posts: 102

#17
(permalink)

A330 with
Para-visual
Indicator (PVI)
which requires
valid LOC
signal for
departure
runway allows
75m RVR for
departure.

Va compared to Rough Air Speed

I fly turbo-props for a living, not been able to get on any jet aircraft yet,
though hopefully should be there soon!
My question concerns the difference between Va and the rough air speed.
Va is almost always lower than rough air speed, why is this? Va is supposed
to allow safety margins above stall at the limit of the airplanes g-loading
envelope, correct? If so why wouldn't this also cover the speed to be flown
in rough, turbulent air when the gust conditions encountered can replicate
the same conditions affecting Va.
I've run across a few chaps who would like the speed flown in turbulence to
be far below the rough air speed in the FCOM so I am curious as to what
should be flown.
Any input would be greatly appreciated.
Thanks!

10th January 2003, 14:08

#2 (permalink)

dolly737

BHS239:

Join Date: Aug 2001


Location: Europe
Posts: 118

Im quite sure
there are
differences in
certification
requirements

(dont have the


regs handy
right now). But
the overall
stress for the
aircraft
(turbulence vs.
manoeuvering)
is NOT the
same.
Manoeuvering
means pilotinduced loads
via the control
surfaces, which
is not
considered in
turbulence, at
least for
certification.
So if you have
a published
turbulence
speed, stick to
it rather than
getting close to
stalling.
happy landings

#3 (permalink)

10th January 2003, 17:31

BHS239
Join Date: Dec 2002
Location: Bahamas
Posts: 6

10th January 2003, 17:42

Tinstaafl
Join Date: Dec 1998
Location: Escapee from
Ultima Thule
Posts: 2,930

10th January 2003, 19:40

Thanks for the reply. being new to the airline system alot of guys
tend to give you thier own made up thoery of flight so to speak,
so a forum of this nature is superb to combat fiction with fact.
Thanks alot for your input.

#4 (permalink)

Va is all about protection by stalling before a maximum load factor


is attained.
Turbulence penetration is concerned with protection from stalling
AND the likelyhood of exceeding maximum load factor from a
gust.

#5 (permalink)

LOMCEVAK
Join Date: Sep 2001
Location: UK
Posts: 252

10th January 2003, 20:18

None
Join Date: Jul 2000
Location: West
Posts: 364

10th January 2003, 23:35

john_tullamarine
Moderator
Join Date: Apr 2001
Location: various places .....
Posts: 3,768

12th January 2003, 21:03

Dick Whittingham
Join Date: Oct 2000
Location: Bristol
Posts: 370

Va is the maximum speed at which full control (rudder,


aileron/spoiler or elevator/tailplane) may be applied without
exceeding the structural limits of the aircraft. The previous post
describes the fundamentals of turbulence penetration speed well.
As has been said above, there is no direct relationship between
the two.

#6 (permalink)

>LOMCEVAK Va is the maximum speed at which full control


(rudder, aileron/spoiler or elevator/tailplane) may be applied
without exceeding the structural limits
It is interesting that you included rudder. In fact, I believe the
definition of maneuvering speed in many books ought to be less
simplistic. However, for the moment, consider that aerodynamic
equations assocoated with maneuvering deal primarily with lift
(G's) and airspeed. It might be found that today many pilots have
re-thought how rudder use fits into these equations.

#7 (permalink)

(a) recent events implicate rudder reversals which are not


addressed in the Standards.
(b) some aircraft have Va limited by rudder considerations in that
directional qualities become a problem .. if my memory serves me
correctly, this was the case with the early 125s.

#8 (permalink)

If we are talking JARs, then all the data is on the JAA site. It is a
copy of FARs, so I suppose the same information is on the FAA
site.
JARs give a fairly basic definition of Va, which is a design speed,
assessed as basic stall speed times the square root of limiting
positive g.
Vb is the design speed for application of gust criteria for
certification. The definition is complex, and is in JARs in full. Vb is
higher than Va. Rough air speed is not a design speed. It is a
recommended speed, chosen to give you the best chance in
turbulence of not hitting either the low speed or high speed buffet
boundaries. Again, JARs explain how it is chosen, but a simple
explanation is that it has to be higher than the minimum
calculated Vb, and can be about half way between Vstall and
Vmo/Mmo.

Although Va, calculated at max. certificated TOW, will not change,


published limiting speeds for manoeuvre or for full control
deflection will change with aircraft weight, as basic stall speed
changes.
But I only quote what was wrote in JARs. John T and others are
your real expert sources.
Dick W

13th January 2003, 12:40

#9 (permalink)

john_tullamarine

The current FAR 23 words may be of use (you need to hyperlink


to 23.335 as this link doesn't want to work as I had hoped) and,
just above 23.335. is a downloadable picture of the envelope
Join Date: Apr 2001
which might assist in understanding. If your penchant is for heavy
Location: various places .....
metal, then the equivalent current FAR 25 words will increase the
Posts: 3,768
level of mental distress once you hyperlink to 25.335 ....
Moderator

The standard caveat applies .. the Standard is an evolving beast


and the present words may be quite different to those which
applied in the past to any particular aircraft model in question. To
resolve this, it is necessary to look to the Type Certificate Data
Sheet for the particular machine and then find the archive version
for the Design Standard.
I think Tinny's description is pretty close to the mark and
generally adequate for pilot purposes ... and certainly a lot easier
than wading through the rule swamp ...
Last edited by john_tullamarine : 13th January 2003 at 12:55.

17th January 2003, 10:14

Volume
Join Date: Dec 2001
Location: what U.S. calls
old Europe
Posts: 355

#10 (permalink)

Va ist the stationary, clean stallspeed for the positive maximum gload at MTOW. If your plane is slower than Va, it will stall before
your g-loads reach the limit value, if your plane is faster than Va,
(or lighter than MTOW) you can exceed the g-limits by applying
full up elevator.
Va is calculated directly from the 1g clean stall speed at MTOW by
multiplying with the square root of the limit gs.
Because Va is the natural limit speed for applying full up elevator,
it is defined also the limit speed for any other full control surface
deflection. (so you design all controls for the given Va value)
Vra is a performance value selected by the manufacturer, without
any direct physical relation to loads or aerodynamics. It must be
choosen smaller or equal to Vc. Vc can also be coosen by the
manufacturer, but it must be greater or equal to an emperical

calculated speed, depending just on the wing loading (JAR/FAR


23, small aeroplanes and commuters) or must provide a safety
margin to Vb (JAR/FAR 25, large aeroplanes). Once you have
choosen Vc, you must use it to calculate gust loads.
Gust loads are not static aerodynamics, so the Cl max value for
the airfoil (and hence the wing) is greater than the static one.
Therefor at Va you can experience gust loads, that are greater
than the limit g-loads, depending on various parameters.
Quite similar to Va, Vb ist the speed at which the maximum
positive gust leads to limit g-loads.
For most transport airplane the (normally assumed) positive 50
ft/s (JAR/FAR 25 : 56 ft/s) gust does NOT exced the g-limits at
Va, so Vb is greater thav Va, but that does not naturally apply to
all aircraft. You calculate your g-loads at Vc for the 50/56 ft/s
gust, which are greater than the Vb limit loads (because Vc is
greater than Vb but the gust is the same), and design your
structure to carry this load.
Afterwards you choose an additional safety margin and define Vra
as the speed, up to which the plane can be flown in strong gusts.
So to answer your original question :
If you want to sell a plane, it must be fast. Therefor you want to
define Vra as large as possible.
On the other hand, you dont want to have to high limit g-loads,
because this makes your plane heavy, so mostly only the
mimimum required value is choosen, leading to a relatively small
Va.
On almost any large plane Vb ist higher than Va, and you choose
Vc even larger. Same applies for the minimum required Vc for
small aeroplanes. And you dont want to choose Vra with too much
nargin to Vc, because you cant sell a slow plane.
So Vc is larger than Va for any conventional plane, and safety
margins dont sell, therefor Vra is also normally higher than Va.

19th January 2003, 01:21

None
Join Date: Jul 2000
Location: West
Posts: 364

#11 (permalink)

Great replies. I appreciate your taking the time to find the


information and the links. Thank you.
Additionally, here is an article that is essentially spot-on.
http://www.aopa.org/asf/asfarticles/2003/sp0301.html

20th January 2003, 00:32

BHS239

#12 (permalink)

I am so pleased with the responses that I am overwhelmed. Still


trying to get it all in. Thanks to all who contributed.

Join Date: Dec 2002


Location: Bahamas
Posts: 6

Chris

#13 (permalink)

24th January 2003, 15:48

Flamgat
Join Date: Aug 2002
Location: Bristol
Posts: 18

Another thought on the subject. If you look at JAR 25.331 on


Flight manoeuvre and gust conditions, only manoeuvre in the
pitching plane is considered. Again in JAR 25.333 the Flight
manoeuvering envelope considers the V-n diagram the emphasis
is on the pitching plane. Although a rolling limit is imposed it is
not the rolling forces that are being considered but the
asymmetric loading of the wings during a pitch manoeuvre with
bank on.
Considering the accident with American Airlines Airbus A300-600R
Flight 587 (facts available on the NTSB web site) perhaps we are
flawed in our conventional thinking that VA is the design
manoeuvering speed for all the controls.
Reading JAR 25.335 on Design Airspeeds carefully, the wording
appears to imply the full application of elevator only.
Perhaps this accident has highlighted the folly of assuming that
VA includes all the aerodynamic controls. If so, a radical change is
needed in our training ,to re-program generations of pilots, to
ensure the next accident is not because some poor mis-informed
pilot has applied full aileron well within VA , only to find the last
round of applause was his wings clapping hand behind his back!

#14 (permalink)

25th January 2003, 00:02

john_tullamarine
Moderator
Join Date: Apr 2001
Location: various places .....
Posts: 3,768

Flamgat,
If you read further on the accident ... you will note that the
concern is not so much with a single, steady application of a
control ... for which the design standards cater .. but with control
input reversals .. for this situation the design process does not
consider structural implications.

#15 (permalink)

25th January 2003, 06:07

411A
Join Date: Mar 2000
Location: Arizona USA
Posts: 7,510

The effects of rapid rudder inputs were well understood in the


1950's and lessons learned well.
THEN enter the jonny-come-lateys at AA who taught maximum
rapid rudder use.
SOME never learn...or forget.

26th January 2003, 19:21

None

#16 (permalink)

What did AA teach? I assumed everyone taught the Boeing/Airbus


recommendations.

Join Date: Jul 2000


Location: West
Posts: 364

27th January 2003, 17:55

BOAC
Per Ardua ad Astraeus
Join Date: Mar 2000
Location: UK
Posts: 11,172

#17 (permalink)

BHS:
<I've run across
a few chaps
who would like
the speed flown
in turbulence to
be far below the
rough air speed
in the FCOM so
I am curious as
to what should
be flown.>
There is
immense
confusion about
speeds to fly in
'turbulence'.
There is
NOTHING wrong
with flying
below Vra in
'turbulence' as
long as the
'turbulence' is
not lifethreatening.
When it is, fly
Vra. Reducing
speed will (?
always? standing by to
be corrected!)
improve the ride
for the salary
payers in light
or even
moderate
chop/turbulence

without
threatening the
aircraft's
structural or
aerodynamic
integrity. When
it looks as if the
bumps are
getting so bad
you think you
will either stall
or break up - fly
Vra - and avoid.

Coming to a full stop before take off

This question might be somewhat ambigious from a technical point of view


- but here goes:
I have been flying a lot lately and have noticed that sometimes the aircraft
starts its roll and the engines start to spool up just as it enters the runway
from the taxiway, while at other times it comes to a full stop on the
runway and then commences take off.
Assuming that the aircraft has not been given a "taxi into position and
hold" clearance - is it preferable to come to a full stop before take off or
not? Also are there specific airline policies that state either?

10th August 2002, 19:01

Intruder
Join Date: May 2000
Location: USA
Posts: 1,707

#2 (permalink)

Different
airlines have
different
policies. Some,
for example,
require a full
stop if the F/O
is the pilot
flying.
Boeing says it
makes no
practical
difference from
a performance
standpoint.

#3 (permalink)

10th August 2002, 19:13

None
Join Date: Jul 2000
Location: West
Posts: 364

It might be preferred to do a rolling takeoff if there is a concern


about FOD. It may also be preferred if there is a concern for
engine acceleration (727 center engine stall/stag prevention for
example).
On the other hand it might be required to do a static engine runup prior to takeoff for certain icing conditions.

#4 (permalink)

10th August 2002, 23:08

fireflybob
Join Date: Jun 2001
Location: Nottingham, UK
Posts: 2,026

Intruder - I am not so sure you are right about what Boeing says.
On the B737-200 they used to say that a rolling take off was
preferred (other than anti ice etc) but if not possible 1.4 EPR must
be set prior to brake release for Perf A etc.
Given current traffic densities at many airports the "stop before
your roll" technique is a luxury which is not often available.

#5 (permalink)

11th August 2002, 00:27

john_tullamarine
Moderator
Join Date: Apr 2001
Location: various places .....
Posts: 3,768

The problem of a rolling start (apart from FOD etc considerations)


is one of acceleration variations.
On a longer runway at higher commercial RTOW, especially for hot
and high conditions, the acceleration is comparatively modest with
the result that the aircraft hasn't gone a great distance before the
spin up puts the engine at takeoff thrust .... in such cases there is
little to worry about.
However, on a short runway at limiting (comparatively light)
RTOW, especially for low and cold conditions, there may be a
measurable problem in that the acceleration at part thrust
settings may be significant. This may then result in the aircraft's
having proceeded some distance down the runway prior to the
spin up reaching takeoff thrust. In such circumstances, there is
the possibility that the rolling takeoff may compromise the sums.
Most operators would get around this problem by doing a few
sums and prescribing a below RTOW tolerance within which more
rigorous attention to technique is required .......
Last edited by john_tullamarine : 11th August 2002 at 02:26.

#6 (permalink)

11th August 2002, 04:19

Capt Claret

Shon

Bottums Up

Join Date: Feb 2000


Location: dunnunda
Age: 52
Posts: 2,464

On the BAe146 with the operator I work for, we have guidance in


the type SOPs (Standard Operating Procedures).
Basically if the aircrafts actual BRW (Brakes Release Weight) is
within 600 kg of the RTOW (Regulated Take Off Weight) we come
to a halt, advance the thrust levers and don't release the brakes
until achieving 60% N1.
If the BRW is within 400 kg of RTOW we must set takeoff N1 prior
to releasing the brake as well as ensuring that we do not
compromise the line up allowance used to calculate the RTOW.
Why?
Basically to ensure that the RTOW is not compromised so that in
the event of an engine failure there is adequate performance for
obstacle clearance.
If say we did a rolling start at BRW = RTOW, it would be quite
possible to have take off thrust set later than the performance
calculations assume. Thus aircraft acceleration would be slower
than predicted and the aircraft would be further down the runway
when it reached V1. A rejected take off could see the aircraft over
run the runway end.
If by spelling out the abbreviations I'm telling you how to suck
eggs, please accept my apology

#7 (permalink)

11th August 2002, 10:08

SLT
Join Date: Dec 1998
Location: UK
Posts: 124

Airbus SOP's suggest a rolling takeoff when possible. I guess the


other thing about this is that at airports where runway occupancy
time is a factor, it would be always preferable to do a rolling
takeoff to keep occupancy time at a minimum to help ATC, even
when you are not given "cleared immediate takeoff". We have all
been given a standard "cleared for takeoff" when there is traffic at
4 miles. Perfectly OK as regards separation, but why reduce the
spacing by stopping??
Our company SOP's across all fleets, and I would imagine in most
other airlines as well is not to accept a line-up or take off
clearance unless you can taxy on and do just that without
stopping.
Hope this helps

#8 (permalink)

11th August 2002, 19:21

Intruder

fireflybob:

Join Date: May 2000


Location: USA
Posts: 1,707

The Boeing info I have is for the 747-400. My intent was to


convey that the standing vs rolling start is not dictated by
runway/performance limitations (i.e., there is no practical
difference or advantage either way for takeoff roll, V1 calculation,
etc). There may well be operational considerations in the decision,
including all those that other people mentioned (FOD, anti-ice,
ATC, etc).

#9 (permalink)

11th August 2002, 21:45

nitro rig driver


Join Date: May 2002
Location: somewhere near
an airport
Posts: 116

i asked this question to a instructor once and the answer was that
boeing says
stopping on the end enables the fuel in the surge tanks and the
top hat section to flow back into the tanks,otherwise it remains
trapped due to the angle of the wing during flight
however i doubt this as they just don/t stay on the end long
enough,but i beleive not all a/c have surge tanks now adays

12th August 2002, 19:46

#10 (permalink)

Flight Detent

Hi 'Nitro rig driver' -- say what!!!

Join Date: Jan 2002


Location: Brisbane,
Australia
Posts: 964

As far as B747 is concerned, it's purely an engine run-up rolling,


verses the run-up static, with less runway used up, when it's
critical, doing the run-up to 1.1 EPR stopped, takes only about 5 8 seconds, and is worth the v/short delay.
Cheers

#11 (permalink)

25th August 2002, 10:42

bsevenfour

Situations when it is better to do a static run up

Join Date: Feb 1999


Location: Asia
Posts: 31

1> In icing conditions it is usually specified.


2> If you have on an aft C of G and low gross weight, conditions
which can lead to very low nose wheel sterring effectiveness.
3> If take off performance is critical. However for aircraft such as
the 747-400 the performance difference between a rolling and a
static take off is minimal.
4> If FOD is a concern.
5> Slippery runway conditions.
On certain engines / aircraft you are also strongly advised NOT to

do a static run up if the crosswind component is greater than 20


kts due to the increased possibility of engine surge.

#12 (permalink)

26th August 2002, 08:47

Flight Detent
Join Date: Jan 2002
Location: Brisbane,
Australia
Posts: 964

Forgive me if I missed something here,


weren't we dicussing rolling take-offs vs stopping prior to actually
commencing the take-off,
a static run-up is a different matter all together, me thinks!
Cheers

#13 (permalink)

31st August 2002, 00:19

crj-jockey
Join Date: Aug 2002
Location: MUC
Posts: 15

Hello,
it allways depends on your calculations and the prevailing
circumstances.
All our T/O calculations are based on a 60m rolling T/O-allowance
(60m rolling until T/O thrust). However, there are certain
circumstances requirering a static T/O(see statemant above).
Nowadays, we do our calculations with a pilots workpad (Laptop),
which allways provide your specific stop margin in event of a RTO.
So the final decision rests with the PIC considering all
prevaivailing circumstances.

2nd September 2002,


20:15

Flight Detent

#14 (permalink)

As it always does !!!

Join Date: Jan 2002


Location: Brisbane,
Australia
Posts: 964

4th September 2002,


14:08

Steamhead
Join Date: Jan 2000
Location:
Wickford,Essex,England
Posts: 88

6th September 2002,


22:03

#15 (permalink)

Try stopping on the runway at LGW on a nice day , with the "A"
team in the tower, and in the rush hour.
It would be enlightning to learn some new swear words.
Regards

#16 (permalink)

GlueBall

Our SOP: Rolling takeoffs, unless otherwise dictated by ATC.

Join Date: Jun 2001


Location: UTC +8
Posts: 2,124

7th September 2002, 11:42

Dan Winterland
Join Date: Jun 2001
Location: Fragrant Harbour
Posts: 2,746

#17
(permalink)

On the 744
(with GE's,
i.e. N1
power ref,
not EPR)
there is no
difference.
On my
previous
type
(VC10), we
used to get
another
tonne's
worth of
RTOW by
stopping
and running
up to 80%
N2 before
releasing
the brakes.

TCAS Question

I was flying with a captain recently and we discussed an incident in which


another aircraft had been vectored to pass 500' over the top of his aircraft.
The incident occurred in the US and the other aircraft was operating VFR.
Aside from the legality of this clearance, what confused us was whether this
would generate a TCAS RA given that both aircraft would pass each other
flying straight and level.
Does TCAS only give RA's if there is an actual impending collision or does it
give RAs if the clearance between you and the conflicting aircraft drops below
a certain figure? If so, what is that minimum clearance?

12th June 2002, 02:04

#2
(permalink)

SimJock
Join Date: Oct 2001
Location: UK
Age: 45
Posts: 173

TCAS has a
range of
sensitivity
bands that
are
switched in
at different
altitudes. It
all depends
what
altitude you
were at as
to whether
you would
get an RA
or not.
Typically,for
2 aircraft
flying level
below
5000ft,
seperation
of 300ft or
less will
generate an
RA,
anything
greater and
a traffic
only
warning will
be given.
Between
5000ft and
10000ft
350ft is min
seperation
rising to
600ft at
20,000ft to
42,000ft.
All this
assumes
level flight
of both
aircraft, if
either starts
climbing or
descending
its a whole

new
ballgame!

#3 (permalink)

12th June 2002, 02:15

SimJock
Join Date: Oct 2001
Location: UK
Age: 45
Posts: 173

I should say that the figures given are those that would generate
a corrective RA (Climb/Descend), there are some other seperation
figures typically within 600ft that would generate preventative
RA's (Don't climb/descend monitor vertical speed) to which no
positive pilot avoiding action is required. Traffic advisories would
occur within 850ft seperation.

#4 (permalink)

12th June 2002, 03:22

411A
Join Date: Mar 2000
Location: Arizona USA
Posts: 7,510

Interesting.
Here I am in my trusy aeromachine at 16,500 feet (VFR) west
bound, flying straight and level, passing over AA proceeding to
PHX at 16,000, and he is having a fit...because the airspace
belongs to him (so he thinks) even tho both of us are talking to
PHX approach.
Needless to say, AA was told..."sorry, but he is at the proper
altitude, call us on the telephone after you land."
Really...some of the airline guys think that the sky belongs to
them, but it certainly does NOT, at least here in the USA.

#5 (permalink)

12th June 2002, 09:46

Young Paul
Join Date: Nov 2001
Location: Inside the M25
Posts: 434

I've had a similar thing (500' separation from a VFR aircraft) in


German airspace.
It always intrigued me that I could apparently be under radar
control below 10000' in German airspace when there were
uncontrolled VFR aircraft also below 10000'. What was I supposed
to do if one tried to hit me? Ask for permission to deviate from
present track?!
[Before somebody says something, don't worry, I wouldn't really
do that.]

#6 (permalink)

12th June 2002, 22:33

None

"guys think that the sky belongs to them"

Join Date: Jul 2000


Location: West

411A, allow me to offer another perspective. When flying within


500 feet vertically of another aircraft who is VFR and may even be

Posts: 364

of an unknown type, it is possible to lose the normal level of


comfort. There may be a question as to how predictable his flight
path is...after all, he is VFR and may change altitude and/or
course at any time. In this case 500 feet is too close. For this
reason one might prefer a vector around such traffic. You may
interpret this as operating at the highest level of safety, or staying
within one's comfort zone, or something otherwise. In any event,
it is another point of view.

411A wrote:
Interesting.
Here I am in my trusy aeromachine at 16,500 feet (VFR) west
bound, flying straight and level, passing over AA proceeding to
PHX at 16,000, and he is having a fit...because the airspace
belongs to him (so he thinks) even tho both of us are talking to
PHX approach.
Needless to say, AA was told..."sorry, but he is at the proper
altitude, call us on the telephone after you land."
Really...some of the airline guys think that the sky belongs to
them, but it certainly does NOT, at least here in the USA.
Last edited by None : 13th June 2002 at 02:08.

#7 (permalink)

13th June 2002, 04:31

411A
Join Date: Mar 2000
Location: Arizona USA
Posts: 7,510

Would certainly agree with you none, except that AA was advised
of the traffic. He did not ask for a vector, just complained that
there was another aircraft near him.
He had the option of course to take avoiding action if he received
an RA, and he had every right to do so.
Unfortunately...as he was AA, he was a "sky God".

#8 (permalink)

13th June 2002, 04:44

AA76757
Join Date: Jul 2001
Posts: 21

And you, 411a, while posessing a huge chip on your shoulder


towards AA, but yet having no clue as to why a professional
airline crew would look twice at 500' separation. Yes it may be the
legal mininum, but no, it isn't a "normal" or "common" situation,
so it does require a second look. It's called airmanship.
Ah, yes, it all makes perfect sense now

13th June 2002, 05:26

ICT_SLB

#9 (permalink)

Generally a Resolution Advisory will only be posted if there is a


probability of collision i.e. if you're at different altitudes there

Join Date: Apr 2002


Location: The Air Capital
Posts: 474

must be some convergent vertical speed.


Don't forget there are two software versions out there. The older
Change 6.04 expects at least 1000 ft seperation and thus is more
likely to bleat if you break that (as in the original post). The
newer Change 7 is designed for RVSM and 500 ft seperation (we
had to get within 300 feet for an alert during some sim testing).
The one thing to remember is that if you have the later software
and get an alert - react immediately - you're definitely on a
collision course if you don't!
This software is produced by Mitre for the FAA so it doesn't matter
who builds your TCAS you'll have the same results.

#10 (permalink)

13th June 2002, 07:57

411A
Join Date: Mar 2000
Location: Arizona USA
Posts: 7,510

Actually AA76757, it is rather common (at least in the PHX, LAS


and LAX areas) especially in the class B airspace.
The AA guys will just have to get used to it.

#11 (permalink)

14th June 2002, 21:46

Spitoon
Join Date: Apr 2002
Location: Europe
Posts: 2,066

I'm getting away from the original question but eeper mentions
the legality of a clearance that puts a VFR aircraft 500ft vertically
away form an IFR flight.
This all depends on the class of airspace. If I recall correctly, in
ICAO terms in Class D or below, there is no requirement to
separate VFR from IFR flights. The only obligation on the
controller is to pass traffic information although here in the UK it
is quite common for controllers to restrict VFR traffic so as to
ensure that there's 500ft vertical between them and an IFR flight.
Specifically on the TCAS question, is it worth mentioning that
you'll only get a RA if the other aircraft is squawking mode C - if
it's mode A only, all you'll get is a TA.
There endeth my knowledge of TCAS!

14th June 2002, 22:02

411A
Join Date: Mar 2000
Location: Arizona USA
Posts: 7,510

#12 (permalink)

Yes, quite correct. However, in class B & C airspace (and in the


30NM veil around class B, mode 'C' is absolutely required in the
USA.
Someone mentioned to me today that mode 'S' would be required
in 2004/5 in class 'B' (USA), can anyone confirm? This may

actually be a good idea as ADB-S works good, at least so far.


Perhaps some tech types here can speculate/confirm.

#13 (permalink)

17th June 2002, 06:55

Tcas climb
Posts: n/a

#14 (permalink)

18th June 2002, 05:23

AlfaMike
Join Date: Jun 2001
Location: Asia.
Posts: 37

this are the requirements in our A/C that may trigger and R/A or a
T/A, hope this helps.
Operating transponder for both A/C.
T/A @ 40 sec. from point of closest app.
R/A @ 25 sec. from point of closest app.
R/A req. at least mode C.
Coordinated R/A req. both TCAS equip.
Alt. and Vert. motion are included if A/C is mode S or C equip.
Prox. TFC advisory if within 6 nm. & 1200 vert.
Vertical motion info is avail if rate > 500/min.

#15 (permalink)

18th June 2002, 21:46

Capt Pit Bull

ICT_SLB

Join Date: Aug 1999


Location: England
Posts: 805

There are (obviously) differences between 6.04 and 7.0, but in


neither type is a vertical rate of closure required in order to
generate an RA.
Both versions have a "Positive RA Threshold" (for corrective RAs)
and a "Preventative RA Threshold" (for preventative RAs). Both
are assessed versus predicted vertical separation at predicted
time of closest approach.
So in all cases you can get RAs, with both aircraft in level flight,
and at a different altitude. It just has to be a difference of altitude
of less than the threshold (which 500', in this example, would be.
A vertical rate of closure is not necessary. In fact, the aircraft
could be vertically diverging and an RA could still be generated.

e.g. - high altitude. Preventative threshold 800', corrective say


700, RA alerting threshold 35 seconds (these figures from top of
head but approx correct). Aircraft currently co altitude. A/C 1 in
level flight. A/C 2 rate of climb 600 fpm. 35 seconds to closest
point of approach - predicted vertical separation 350'. Result Corrective RAs for both aircraft (if equipped).
Spitoon
Sorry to pick hairs, but thats 'altitude reporting' rather than 'Mode
C'.
TCAS can not see Mode A only (it doesn't send mode A
interrogations), but what people refer to as Mode A only
transponders are actually Mode A and C without altitude
reporting.
CPB

#16 (permalink)

18th June 2002, 22:42

Denti
Join Date: Mar 2001
Location: In some hotel
downroute or in some hotel
doing union negotiations.
Posts: 909

According to Honeywell TCAS can see Mode A-answers. Just read


the article.
Denti

#17 (permalink)

19th June 2002, 17:55

Capt Pit Bull

Denti,

Join Date: Aug 1999


Location: England
Posts: 805

it says 'Mode A and C transponders'.


Note this means:
(Mode A and C) transponders,
as opposed to:
Mode A transponders AND Mode C
transponders,
or:
Mode A transponders OR Mode C
transponders.

When we say Mode A and C transponders


we are talking plural examples of ONE
type of tranponder i.e. 'A and C', as
opposed to plural types of tranponders.
There is no such thing (anymore) as a
solely Mode A transponder, or a solely
Mode C transponder, only Mode "A and C"
(or "S").
The distinction is one of altitude reporting
or not.
If your transponder does not have altitude
encoding, (or altitude reporting is turned
off) it will still reply to mode C
interrogations, just with an empty pulse
frame.
For a fuller explanation see this thread
from a few months back.
TCAS Questions
CPB
[edited to fix link]
TCAS system

Hi all,
I'm doing an important uni project, and need to
find out about the TCAS Traffic-Alert and
Collision Avoidance System. I understand that
TCAS I works using the Mode C transponder, but
I have read that TCAS II requires Mode S. Is this
true, or can TCAS II run using only Modes A &
C?
I am also still unsure as to how Mode S really
work - I have read so much, that I have started
to confuse myself
. Any simplified explanation
would be apprecaited.
Finally, I need to know whether TCAS III (with
the horizontal resolution advisories) also needs
Mode S, and if so, does is still need A & C?

Cheers in advance for your time

Phill T

#2 (permalink)

12th April 2002, 20:10

Speedbird252

Hi Phil, go to the link below, when you get


there, do a search on TCAS and it will
take you to their TCAS pages, and you will
find everything you ever wanted to know
about it. The latest systems, what they do
and how they do it. There is a
downloadable manual too which is a good
read.

Posts: n/a

Hope it is of help.
Regards,
Woop woop traffic.........
http://lists.arinc.com/products/index.html

#3 (permalink)

13th April 2002, 10:08

spoilers yellow
Join Date: Feb 2001
Location: Heathrow
Posts: 145

As i understand it, in order for an aircraft to be able to resolve a


possible conflict both aircraft must be mode S equipped.
To have an indication of aircraft altitude on your own TCAS display
the other aircraft must be at least mode C equipped. if it has no
mode C ie mode A only you will get a symbol of an aircraft
generated in the position it is in relation to your self but no
altitude info and therefore no conflict alerting.
As for TCAS III, I think (and correct me If i am wrong) that it is
still in test phases and I am not sure that it has been fitted to any
commercial airliners as yet due to the accuracy levels it requires.

#4 (permalink)

13th April 2002, 15:51

RadarContact

Small correction here:

Join Date: Feb 2002


Location: Planet Earth
Posts: 261

You will get an alert when approaching a target with mode A.


What you won't get is a resolution advisory.

#5 (permalink)

13th April 2002, 16:59

411A
Join Date: Mar 2000
Location: Arizona USA
Posts: 7,510

....and when approaching another aircraft with only mode C (S


not required) you should receive an RA.....had a few bugs in the
beginning...but now works good.
Another nice piece of equipment that was invented in
America....along with GPS, INS, VOR, LORAN, ILS...well the list
goes on. But just to show that the British came up with a few
good ideas...RADAR, turbine engines, and oh yes, the Comet.

#6 (permalink)

13th April 2002, 17:14

Doctor Cruces
Join Date: Feb 2000
Location: UK
Posts: 345

Yes we Brits do invent good things. Unfortunately if the Americans


are a bit behind they try to kill it off, like the aformentioned
Comet, Concorde springs to mind as well.
(Sound of can of worms opening!!)
Doc C.

#7 (permalink)

14th April 2002, 16:00

Capt Pit Bull

Philltowns,

Join Date: Aug 1999


Location: England
Posts: 805

This confusion about the issue of whether


TCAS II needs mode S to work or not
results from the following:
In order for TCAS II to work in an
aircraft, that aircraft must be equipped
with at least 1 functioning Mode S
transponder.
Once an aircraft is equipped, the TCAS
can generate various advisories against
any intruder that is equipped with an
ICAO compliant transponder. In all cases,
traffic advisories (TAs) can be generated.
When the intruder is altitude reporting
(be it Mode S or Mode C) then a
Resolution Advisory (RA) is issued when
necessary. When the intruder is TCAS
equipped, the RAs generated are
coordinated, by datalinking via the mode
S transponders, to ensure that the
avoiding manoeuvres are coordinated
(e.g. to make sure one aircraft pitches
up, and the other down, rather than both
up!).

I.E. *You* have to have mode S for your


TCAS to work. The intruder doesn't.
The other area of confusion about this
issue is what is meant by a Mode C
transponder versus a Mode A
transponder.
Mode A is an interrogation sequence that
prompts the transponder to reply by
encoding its identifying 4096 code.
Mode C is an interrogation sequence that
prompts the transponder to reply by
encoding its altitude.
Hence, most people (and most
textbooks) take the view that when an
aircraft is not equipped with altitude
encoding it is equipped with a Mode A
rather than a Mode C transponder. This is
incorrect.
ALL non mode S transponders reply to
both Mode A and Mode C interrogations.
ATC radars send out both a Mode A and a
Mode C interrogation. When the aircraft it
not equipped with Altitude reporting, the
transponder still replies to the mode C
call, but with a pulse frame only, and the
encoded section of the reply is omitted.
I.E.
What is popularly referred to as a 'Mode
A' transponder is in fact a 'Mode A and C
without Altitude Reporting transponder'.
What is popularly referred to as a 'Mode
C' transponder is in fact a 'Mode A and C
with Altitude Reporting Transponder'.
This incorrect designation even goes as
far as many transponder controllers,
where the mode selecting knob is labelled
something like 'Off', 'STBY', 'A', 'A+C',
when it should be 'Off', 'STBY','On',
'On+Alt'.
Nevertheless, I accept that this is a fine
distinction that does not really affect a
typical line pilot. However, if you want to

understand how TCAS works, then it


becomes an issue......
Because practically every nuance of how
TCAS and Mode S function is driven by
the engineering need to avoid any
unnecessary transmissions in the
secondary radar bands. Whereas preTCAS, the only transmissions were from
ground radar heads to aircraft, now every
aircraft is also keeping track of every
other aircraft within the vicinity
(something like 40 NM). Therefore,
without care, the entire secondary radar
bands would be swamped!
Mode S includes a lot of tricks to achieve
this, but I don't propose to talk about it
in this post but will expand if you wish.
As far as other transponders go,
appreciate that TCAS doesn't care what
4096 ident code the intruder is wearing.
It only cares about the range to the
intruder, and the intruders altitude.
To find the the range, a time delay versus
speed of light calculation is required. This
simply needs the intruders transponder
to reply, and this could be achieved by
either a mode A or a Mode C
interrogation.
To find the altitude, obviously a Mode C
interrogation is required.
From this we can see that if TCAS sends
a Mode C interrogation, it can achieve
both objectives, both range and Altitude.
Therefore there is no need for TCAS to
send a Mode A interrogation, so to avoid
garbling the secondary radar frequencies
it doesn't.
I.E. TCAS only sends mode C, and not
mode A, interrogations for dealing with
non mode S intruders,
So, technically, TCAS will only see Mode
C and Mode S transponders.
So, technically, TCAS will not see Mode A
transponders, and you will not get any

advisories.
However, what people think are Mode A
transponders are in fact Mode A and C
transponders, just without altitude
encoding.
I hope that helps...
CPB
TCAS instructor
Calculating cloud heights with radar?

Dusting off the cob webs on radar usage, does anyone remember a
formula for calculating cloud tops using tilt and distance?
Thanks.

#2 (permalink)

30th August 2001, 12:24

twistedenginestarter
ENTREPPRUNEUR

Isn't this yer ole


one-in-sixty?
Thus the cloud
tops would be:

Join Date: Jun 2001


Location: The 60s
Posts: 513

tilt X nautical
miles X 100
This of course
has to be
added/subtracted
to/from your
current altitude.

#3 (permalink)

30th August 2001, 13:56

Captain Airclues
Just another number
Join Date: Jun 2001
Location: UK
Age: 62
Posts: 1,087

PFM
Just get a glass of water and look along the surface of the water.
If you can see the cloud you'll hit it, if you can't you won't.
Airclues

PS. The clever people will tell you about the water meniscus and
the curvature of the earth, but it's worked for me for 35 years.
PPS. You do get some strange looks from the F/O though

#4 (permalink)

30th August 2001, 14:27

5150
Join Date: Apr 2000
Location: UK
Posts: 561

Radio Nav theory recommends the following equation for the ATPL
writtens;
(Tilt Angle - 0.5 beamwidth) x range (nm) x 6080 = Answer in
feet.
Answer, like above, needs to be applied to current altitude.

#5 (permalink)

30th August 2001, 14:32

Manflex55
Join Date: Jun 2001
Location: France & UK
Posts: 882

As twisted said, the 1 in 60 rule works fine. To determine the


cloud ht, the wx beam is raised by turning the tilt knob until the
cloud just disappears from the screen. Let's say our beam has a
width of 5:
- if the cloud disappears as tilt setting reads, for example, "4
UP" (which means a beam centreline 4 above the horizontal),
then the beam's LOWER line (& hence the cloud top) is 2 above
your current alt.
- should U B, for example, 45nm away, using the 1 in 60 rule U
get a cloud top of 9,000' above your current alt.... so plenty of
time to climb & pass above it. Don't forget the recommended
2,000' margin if your cloud is a CB !!
MF

30th August 2001, 15:16

#6 (permalink)

fireflybob
Join Date: Jun 2001
Location: Nottingham, UK
Posts: 2,026

Funny but I thought wx radar showed significant amounts of


"precipitation" as opposed to cloud?
This seems to be the latest favourite interview question!

#7 (permalink)

30th August 2001, 15:31

Checkboard
Join Date: Aug 1998
Location: Ex-pat Aussie in
the UK
Posts: 1,408

That's true, the above formulae give the radar height of the cloud
- the top of detectable precipitation. The actual cloud will extend a
third or so higher, depending on the cloud.
If I feel keen tomorrow, I will include an article on radar I have
filed somewhere.
In the meantime, have a read of these two articles from the
TechLog Archive:
Terminal Area Weather Radar Technique, and
X band radar tips?

#8 (permalink)

30th August 2001, 15:49

dv8
Join Date: May 2001
Location: Location Location
Posts: 325

No pilot manual of any A/C I have flown have dared to tell me the
beam width of the AWR.
Hush Hush, need to know and all that.

30th August 2001, 22:40

#9 (permalink)

411A
Join Date: Mar 2000
Location: Arizona USA
Posts: 7,510

4th September 2001,


03:06

In NO aircraft that I have flown in the last 35+ years has the
radar tilt indication been especially accurate.
As with the glass of water routine, many years ago flew with
Captain who carried a small bubble level to sight along to
estimate cloud tops. Seemed to work for him anyway.

#10 (permalink)

LightenupFrancis
Join Date: Jan 2001
Location: AZ
Posts: 25

I agree with what Manflex55 said regarding the tilt and 60 to 1


rule. I take it one step further by remembering that the earth is
round and its horizon constantly falling. Hence, for every 60nm
away a return is, you subtract 1 deg of tilt from the equation. 60

x 360 = 21600nm (pretty close to the earth's circumference). So


a storm I paint with precip tops of 30,000ft at 30nm with a
hypothetical tilt of 2 1/2 deg up bottom of the beam,
would/should show up with 1 1/2 deg up tilt at 90nm. If it shows
up at 90nm with the same tilt, it would be 6000ft higher, or
36,000ft. I used to fly a lot of 10hour flights, what else was I
supposed to do with my mind?

#11 (permalink)

6th September 2001, 12:17

twistedenginestarter
ENTREPPRUNEUR
Join Date: Jun 2001
Location: The 60s
Posts: 513

kelbill
Registered January but this is your first post. Glad to see you lost
your viginity on Tech Log where the elite people hang out.
Well, welcome. And let's hope we don't have to wait so long for
Post No 2.

#12 (permalink)

5th November 2001, 21:02

ITCZ
Join Date: Jan 2001
Location: Australia
Posts: 716

411A, you are right about the tilt calibration -- rarely will the
"indication" equal the actual tilt.
Easiest way to "calibrate" it yourself for each flight is to use the
100'/1nm rule of thumb. Tilt down till ground return = height
(not altitude AMSL) in thousands. Eg F350, closest ground return
@ 35 miles. Bottom of beam is 10 degrees down. You do the
maths.
Generally speaking, according to company 'coneheads' a 10 inch
diameter wx radar dishes produce a beam 8 degrees wide, and
the beam width reduces by one degree for each increase of one
inch diameter of the dish. Generally speaking. Writer makes no
warrantee as to the accuracy of above statement. Break the habit
of a lifetime and talk to a radio tech yourself before you sledge
me. Even better -- ask the radtech to find out the beamwidth for
your installation from the bucketload of manuals he/she has.
Armed with those two slightly dodgy bits of info, you can then do
some more 1/100 with your tilt in flight and come up with handy
guesstimates. Not accurate to the millimeter, but we don't give
our ETA to the decimal of a second either.
[ 05 November 2001: Message edited by: ITCZ ]

#13 (permalink)

6th November 2001, 06:10

Checkboard
Join Date: Aug 1998
Location: Ex-pat Aussie in
the UK
Posts: 1,408

Antenna type vs dish size & beam width


Size
10"
12"
18"
24"
28"
30"

X-band
10
8
5
4
3.5
3

C-Band
NA
NA
NA
6.5
6
5

(From "X band radar tips?", posted above.)


[ 06 November 2001: Message edited by: Checkboard ]

#14 (permalink)

9th November 2001, 04:39

Agaricus bisporus
Join Date: Dec 1999
Location: UK
Posts: 1,091

PFM, if you'll provide an e-mail address I can mail you a 4 page


article on advanced radar techniques which covers this and many
other systems for scientific use of radar. Originally published in an
American aviation mag some years ago, very useful.

10th November 2001,


18:21

#15 (permalink)

4dogs
Join Date: Jun 1999
Location: Australasia
Posts: 201

Agaricus,
Would you mind mailing me a copy as well, please?

10th November 2001,


22:42

#16 (permalink)

Tee
Join Date: Jun 2001
Location: United Kingdom
Posts: 123

The formula I was given for the Radio Nav ATPL writtens is:(tilt - half beam width) x range (nm) x 101.3

101.3 is a constant

#17 (permalink)

11th November 2001, 23:43

Agaricus bisporus
Join Date: Dec 1999
Location: UK
Posts: 1,091

Sorry chaps, if I'd read the above posts


properly I'd have seen the article in
Checkerboard's link. It's the Trammell
terminal area radar article, very useful, and
as long as you know the diameter of your
antenna it is independant of tilt calibration, ie
it is self calibrating.Thoroughly
reccommended.

Why does a light aircraft descend faster than a heavy one?

I still can't figure it out. Why does the B737 sink faster when its light
versus when it is heavy, using a constant speed. I would imagine that to
maintain the same speed during descent, a heavy aircraft would need
less angle of attack thus decreasing its ROD but it is not very clear to
me. Is this a correct assumption? Anyone who could explain this one to
me??
Thanks,
MD

#2 (permalink)

4th September 2001, 21:53

mutt
Join Date: Sep 1999
Location: ME
Posts: 3,622

At any constant high speed, the rate of


descent is higher for light weights than for
heavy weights. This is a function of the
lift/drag ratio.

Mutt
[ 04 September 2001: Message edited by:
mutt ]

4th September 2001,


23:21

#3 (permalink)

Bellerophon
Join Date: Dec 2000
Location: UK
Posts: 183

Mad Dog 11
An aircrafts best glide angle is found at max L/D, which does not
vary with weight but is purely a function of the design and
aerodynamic efficiency of the aircraft.
A variation in weight however does affect the speed at which the
max L/D occurs, the best glide speed increasing as the aircraft
weight increases.
If we consider two aircraft, identical apart from their weights,
both trying to achieve the best glide angle possible in their
descents, we will find that the heavier aircraft, flying at its best
glide speed (max L/D) will be flying faster than the lighter aircraft,
which can still achieve the same glide angle, but only at a slower
speed.
In practice, the best glide speed for both aircraft is likely to occur
at too slow a speed for schedule regularity or ATC purposes etc.,
and so many operators have a fixed speed in descent for their
aircraft, regardless of weight, and this fixed speed is generally
well above the theoretical best glide speeds of either aircraft.
Let us assume that our airline requires us to descend at 300 kts
IAS, and that the best glide speed for our heavier aircraft is 270
kts and our lighter aircraft is 250 kts.
Both could achieve exactly the same glide performance if allowed
to fly at their varying best glide speeds, but they have both been
made to fly at the same speed.
We can see that at 300 kts the heavier aircraft is much nearer to
its best glide speed (only 30 kts too fast) than the lighter aircraft
(50 kts too fast). Being closer to its best glide speed will mean a
better glide performance, and so our heavier aircraft will lose less
height over a given ground distance than the lighter aircraft. As
both are flying at the same speed, this means that the heavier
aircraft has a lower rate of descent than the lighter one!
This ability to increase the speed at which max L/D occurs by
increasing the aircraft weight is used to good effect by
competition glider pilots, who, on good thermalling days, will
often load up to 500 lbs of water into their gliders, but thats
another story!
Hope this helps!
Bellerophon

5th September 2001,


00:19

#4 (permalink)

Ace MCcoy
Posts: n/a

Guys, you make it a joy to browse these pages!


Belle...you MUST be teaching, when not in the sim, preparing for
the big day?? Assuming you are spending a lot of time in the sim
now of course

5th September 2001,


03:16

#5 (permalink)

m&v
Join Date: Dec 1999
Location: delta.bc.canada
Posts: 259

The heavier weight develops more enery going downhill than the
lighter Aircraft.More energy/speed more lift-lower rate of descent.
Lighter aircraft doesn't develope the same speed(glides down),but
better range for time.
Both aircraft would reach the ground 'same place'but different
times.

5th September 2001,


03:59

#6 (permalink)

jtr
Join Date: Aug 2000
Location: .
Posts: 644

5th September 2001,


06:03

Bellerophon, great explanation!

#7 (permalink)

chris barlow
Join Date: Sep 2001
Location: Boston
Posts: 5

In actuality an aircraft will glide at constant dynamic pressure. If


we assume that all glides are performed at the best L/D and that
the aerodynamice are linear, then all glides will have the same
best L/D angle of attack and occur at the same best L/D lift
coeffieient. This is independent of the weight. The flightpath is
also invariant with weight and is given by the L/D ratio. An
increased weight at the same lift coefficient and flight path angle
will require a proportional increase in dynamic pressure to support
it. Therefore an increase in weight should produce a faster speed
down the same glideslope.

5th September 2001,


08:05

#8 (permalink)

Capt Claret
Bottums Up

Join Date: Feb 2000


Location: dunnunda
Age: 52
Posts: 2,464

Mad Dog
Another way to look at it is:
Max glide range occurs at the best L/D ratio AoA. This AoA doesn't
change with weight, thus the weight of the aircraft doesn't change
the glide range.
However, the speed required to achieve this AoA does vary with
weight. In the 146 this speed is defined as Vfto (min speed with
zero flap) + 30 kts. Vfto also varies with weight, the higher the
weight, the higher the value of Vfto.
So if on descent you are say at MLW, you have the highest value
of Vfto. If descent is flown at 250 kts, the difference between 250
kts and Vfto is smaller than say if you are several tonnes lighter,
with a comensurately lower Vfto.
As a jet's descent is essentially a glide with minimal/partial power,
the IAS flown is controlled by (the secondary effect of) pitch
attitude.
As weight decreases, Vfto decreases, the split between descent
IAS and Vfto increases and, the attitude required to convert the
potential energy (of altitude) into the kinetic energy required to
maintain decsent IAS must decrease (lower pitch attitude), thus
increasing the RoD.

#9 (permalink)

5th September 2001, 12:28

twistedenginestarter
ENTREPPRUNEUR
Join Date: Jun 2001
Location: The 60s
Posts: 513

Capt Claret
Quote:

This AoA doesn't change with weight


Oh but it does.
5th September 2001, 12:43

twistedenginestarter
ENTREPPRUNEUR

#10 (permalink)

Join Date: Jun 2001


Location: The 60s
Posts: 513

Here's my position.
To maintain flight the aeroplane must generate lift. To generate
lift it incurs drag and thus must provide thrust.
A heavier plane must increase its angle of attack to provide extra
lift. It thus incurs proportionately more drag and thus requires
proportionately more thrust.
The thrust comes from the vertical component of the glide ie the
rate of change of height times the weight of the plane.
So the heavier plane needs more thrust because it is heavier. But
it is heavier so it provides more thrust for a given angle of
descent.
What I am saying here is you don't have to 'pay' more to glide a
heavy plane. You have already paid for the weight when you
climbed.
This leaves you at the mercy of the wing designer as to whether
you will glide better or worse at higher wing loads. Your 737 may
do better heavy. Another plane might do better light.

5th September 2001,


15:28

#11 (permalink)

chris barlow
Join Date: Sep 2001
Location: Boston
Posts: 5

5th September 2001,

If we're talking pure glide then then there is no thrust. For a given
aircraft configuration, the best glide range will occur at a fixed
angle of attack at which the L/D ratio is a maximum. At this fixed
aoa, the lift coefficient is constant. The extra lift force required for
the heavier weight is supplied by the increased dynamic pressure
and not by an increase in AOA. I'm not sure this really answers
the original post , which concerned itself with constant speed
descents. At constant aoa, the glide dynamic pressure is constant,
however the glide velocity will decrease as the air density
increases at lower altitudes during the glide. To maintain constant
velocity during the descent, the aoa must be varied such that the
required dynamic pressure increases in proportion to the increase
in density during the glide. This will produce a constant speed
glide. Note that the glide range will no longer be the best glide
range for the aircraft and that the flight path angle will be
steepening during the glide.

#12 (permalink)

15:30

1.3VStall
Join Date: Jun 2000
Location: England
Posts: 218

Chaps,
Belle is absolutely spot on. An aircraft's L/D ratio (i.e. glide angle)
is not affected by weight. However, for a given aerodynamic
configuration, as the aircraft weight increases the speed at which
the best L/D is achieved also increases.
This is precisely why we glider pilots carry hundreds of pounds of
water ballast in our high-performance sailplanes on good crosscountry soaring days. We are able to fly at higher inter-thermal
speeds more efficiently.

5th September 2001,


18:07

#13 (permalink)

Mad Dog 11
Join Date: Mar 2000
Location: SP, Brasil
Posts: 18

This is the reason why I ask my questions in this forum. Thank


you all for the replies.
MD

10th September 2001,


12:57

#14 (permalink)

go with the flow


Join Date: Jul 2001
Location: solaris
Posts: 40

A marginal point but doesn't skin drag increase with speed at a


different order to the directly proportional increase in work
"gained" in falling/gliding at increased speed? Therefore won't a
heavier but otherwise identical aeroplane although flying at the
same AoA and theoretical glide path actually fall slightly short as a
result of greater drag?

11th September 2001,


02:12

#15 (permalink)

twistedenginestarter
ENTREPPRUNEUR
Join Date: Jun 2001
Location: The 60s
Posts: 513

Look chaps. Lot's of people are watching us. So let's not make
elementary mistakes.
The lift on a wing is a function of

wing shape
angle of attack
air density
freestream velocity
wing surface area

Lift = Cl 1/2 p v **2 S


Now you can see the only variables are angle of attack and
speed. Unless you increase speed you must increase angle of
attack to maintain the same glide angle if weight increases.
[ 10 September 2001: Message edited by: twistedenginestarter ]

11th September 2001,


09:11

#16 (permalink)

Ignition Override
Join Date: Jul 2000
Location: Down south, USA.
Posts: 1,599

How does this apply to the real world?


In an old airplane at FL 310 with almost no tailwind or headwind,
BUT without VNAV, you are at normal cruise speed about 100
miles from a VOR and you are given a clearance to cross it at
12,000' and 250 knots. The descent should have no more than
light turbulence (negating the need to fly at "rough air speed",
i.e. .79 or 285 knots). Are you going to descend earlier with a jet
weighing about 98,000 lbs compared to one at about 82,000 lbs,
and if so, about how many miles earlier? Roughly a normal
two/one ratio (i.e. double your altitude to lose and add on miles
for speed reduction and several extra miles)?
Does most such study material in various countries apply to line
flying? Just curious.
[ 11 September 2001: Message edited by: Ignition Override ]

11th September 2001,


15:15

#17 (permalink)

Allotta Fagina
Join Date: Jul 2001
Location: Australia
Posts: 5

Dearest El homoez,
Mutt, Maddog11, Ace Mc Coy & Missle man(parlez vous l'anglais
"CTUALITY")

Are you really that stupid or are you just being a f%c&ing tosser
you no life sh*t....
wake up to yourself you idjit...

You might also like